4410 exam Practice Questions

¡Supera tus tareas y exámenes ahora con Quizwiz!

The nurse is caring for a patient admitted with a history of hypertension. The patient's medication history includes hydrochlorothiazide daily for the past 10 years. Which parameter would indicate the optimal intended effect of this drug therapy? 1 Weight loss of 2 lb 2 BP 128/86 mm Hg 3 Absence of ankle edema 4 Output of 600 mL per 8 hours

2 Hydrochlorothiazide may be used alone as monotherapy to manage hypertension or in combination with other medications if not effective alone. After the first few weeks of therapy, the diuretic effect diminishes, but the antihypertensive effect remains. Because the patient has been taking this medication for 10 years, the most direct measurement of its intended effect would be the blood pressure.

A client is considered to be in septic shock when what changes are assessed in the client's labwork? 1 Blood glucose is 70-100 mg/dL 2 An increased serum lactate level 3 An increased neutrophil level 4 A white blood count of 5000 cells/µL

2 The hallmark of sepsis is an increasing serum lactate level, a normal or low total WBC count > 12,000 cells/µL or < 4,000 cells/µL and a decreasing segmented neutrophil level with a rising band neutrophil level. Blood glucose levels with sepsis are between 110 and >150 mg/dL. Blood glucose levels of 70-100 mg/dL are considered normal.

The nurse is caring for a 73-yr-old male patient with a history of benign prostatic hyperplasia and symptoms of a urinary tract infection. Which diagnostic finding would support this diagnosis? 1 White blood cell count is 7500 cells/µL. 2 Antistreptolysin-O (ASO) titer is 106 Todd units/mL. 3 Glucose, protein, and ketones are present in the urine. 4 Nitrites and leukocyte esterase are present in the urine.

4 A diagnosis of urinary tract infection is suspected if there are nitrites (indicating bacteriuria), white blood cells (WBCs), and leukocyte esterase (an enzyme present in WBCs indicating pyuria). The presence of glucose and ketones indicate uncontrolled diabetes mellitus. An elevated WBC count (>11,000 cells/µL) indicates a bacterial infection. AASO titer is a blood test to measure antibodies against streptolysin O, a substance produced by group A Streptococcus bacteria.

A 21-yr-old female patient came to the clinic for instruction to prevent recurrence of urinary tract infections. Which patient statement indicates that teaching was effective? 1 "I will urinate before and after having intercourse." 2 "I will use vinegar as a vaginal douche every week." 3 "I should drink three 8-oz glasses of water daily." 4 "I can stop the antibiotics when symptoms disappear."

1 The woman should empty her bladder before and after sexual intercourse. She should avoid vaginal douches and maintain adequate oral fluid intake (15 mL per pound of body weight). All of the antibiotics should be taken as prescribed even if symptoms are no longer present.

The nurse is examining the electrocardiogram (ECG) of a patient just admitted with a suspected MI. Which ECG change is most indicative of prolonged or complete coronary occlusion? 1 Sinus tachycardia 2 Pathologic Q wave 3 Fibrillatory P waves 4 Prolonged PR interval

2 The presence of a pathologic Q wave, as often accompanies STEMI, is indicative of complete coronary occlusion. Sinus tachycardia, fibrillatory P waves (e.g., atrial fibrillation), or a prolonged PR interval (first-degree heart block) are not direct indicators of extensive occlusion.

The nurse teaches a 28-yr-old man newly diagnosed with hypertension about lifestyle modifications to reduce his blood pressure. Which patient statement requires reinforcement of teaching? 1 "I will avoid adding salt to my food during or after cooking." 2 "If I lose weight, I might not need to continue taking medications." 3 "I can lower my blood pressure by switching to smokeless tobacco." 4 "Diet changes can be as effective as taking blood pressure medications."

3 Nicotine contained in tobacco products (smoking and chew) cause vasoconstriction and increase blood pressure. Persons with hypertension should restrict sodium to 1500 mg/day by avoiding foods high in sodium and not adding salt in preparation of food or at meals. Weight loss can decrease blood pressure between 5 to 20 mm Hg. Following dietary recommendations (e.g., the DASH diet) lowers blood pressure, and these decreases compare with those achieved with blood pressure-lowering medication.

While auscultating the heart, a healthcare provider notices S3 heart sounds in four clients. Which client is at more risk for heart failure? 1 Child client 2 Pregnant client 3 Older adult client 4 Young adult client

3 The S3 is the third heart sound heard after the normal "lub-dub." It is indicative of congestive heart failure in adults over 30 years old. In young, pregnant, and under 30 year old clients, the third heart sound is often considered to be a normal parameter.

An older client who has had multiple hospital admissions for recurring heart failure is returned to the hospital by an adult child. The client is admitted for observation to the coronary care unit and calmly states, "I know I'm sick, but I can really take care of myself at home." What should the nurse conclude that the client most likely is attempting to do? 1 Suppress fears 2 Deny the illness 3 Maintain independence 4 Reassure the adult child

3 The client's statement is really saying, "I can manage this myself. I am capable." None of the information given leads to the conclusion that the client is suppressing fears. Nothing in the statement can be interpreted as denial; the client has stated, "I know I'm sick." Telling the adult child that self-care is possible will not be reassuring to a family member who brought the client to the hospital and who probably is more reassured by having the client hospitalized.

Which antilipemic medications should the nurse question for a patient with cirrhosis of the liver (select all that apply.)? 1 Niacin 2 Cholestyramine 3 Ezetimibe (Zetia) 4 Gemfibrozil (Lopid) 5 Atorvastatin (Lipitor)

3,4,5 Ezetimibe (Zetia) should not be used by patients with liver impairment. Adverse effects of atorvastatin (Lipitor), a statin drug, include liver damage and myopathy. Liver enzymes must be monitored frequently and the medication stopped if these enzymes increase. Niacin's side effects subside with time, although decreased liver function may occur with high doses. Cholestyramine is safe for long-term use.

The nurse instructs a 68-yr-old woman with hypercholesterolemia about natural lipid-lowering therapies. The nurse determines further teaching is necessary if the patient makes which statement? 1 "Omega-3 fatty acids are helpful in reducing triglyceride levels." 2 "I should check with my physician before I start taking any herbal products." 3 "Herbal products do not go through as extensive testing as prescription drugs do." 4 "I will take garlic instead of my prescription medication to reduce my cholesterol."

4 Current evidence does not support using garlic in the treatment of elevated cholesterol. Strong evidence supports the use of omega-3 fatty acids for reduction of triglyceride levels. Many herbal products are not standardized and effects are not predictable. Patients should consult with their health care provider before starting herbal or natural therapies.

Which urinalysis finding indicates a urinary tract infection? 1 Presence of crystals 2 Presence of bilirubin 3 Presence of ketones 4 Presence of leukoesterase

4 Leukoesterases are released by white blood cells as a response to an infection or inflammation. Therefore, the presence of this chemical in urine indicates a urinary tract infection. The presence of crystals in the urine indicates that the specimen had been allowed to stand. Presence of bilirubin in the urine indicates anorexia nervosa, diabetic ketoacidosis, and prolonged fasting. The presence of ketones indicates diabetic ketoacidosis.

A patient with type 2 diabetes is reporting a second urinary tract infections(UTI)within the past month. Which medication should the nurse expect to be ordered for the recurrent infection? 1 Ciprofloxacin 2 Fosfomycin 3 Nitrofurantoin 4 Trimethoprim-sulfamethoxazole

1 This UTI is a complicated UTI because the patient has type 2 diabetes, and the UTI is recurrent. Ciprofloxacin would be used for a complicated UTI. Fosfomycin, nitrofurantoin , and trimethoprim-sulfamethoxazole should be used for uncomplicated UTIs.

Hypertension develops in a school-aged child with acute glomerulonephritis. What medication does the nurse anticipate that the healthcare provider will prescribe? 1 Digoxin 2 Furosemide 3 Diazepam 4 Phenytoin

2 Furosemide inhibits the reabsorption of sodium and chloride from the loop of Henle and distal tubule, increasing urine output and thereby decreasing blood pressure. Digoxin increases the contractility and output of the heart; it is not an antihypertensive. Diazepam is inappropriate; it relaxes skeletal muscle, not the smooth muscle of the arterioles. Phenytoin is an anticonvulsant; it does not reduce blood pressure.

In caring for a patient admitted with poorly controlled hypertension, which laboratory test result should the nurse understand as indicating the presence of target organ damage? 1 Serum uric acid of 3.8 mg/dL 2 Serum creatinine of 2.6 mg/dL 3 Serum potassium of 3.5 mEq/L 4 Blood urea nitrogen of 15 mg/dL

2 The normal serum creatinine level is 0.6 to 1.3 mg/dL. This elevated level indicates target organ damage to the kidneys. The other laboratory results are within normal limits.

Despite a high dosage, a male patient who is taking nifedipine (Procardia XL) for antihypertensive therapy continues to have blood pressures over 140/90 mm Hg. What should the nurse do next? 1 Assess his adherence to therapy. 2 Ask him to make an exercise plan. 3 Instruct him to use the DASH diet. 4 Request a prescription for a thiazide diuretic.

1 A long-acting calcium-channel blocker such as nifedipine causes vascular smooth muscle relaxation, resulting in decreased systemic vascular resistance and arterial blood pressure and related side effects. The patient data the nurse has about this patient is very limited, so the nurse needs to begin by assessing adherence to therapy.

A client is admitted to the emergency department with chest pain and shortness of breath. An electrocardiogram indicates that the client is experiencing a myocardial infarction. An emergency cardiac catheterization is scheduled. What information should the nurse include in the preprocedure teaching? 1 Mild sedation is maintained during the procedure. 2 The procedure will take approximately 15 minutes to complete. 3 Ambulation is encouraged shortly after the procedure. 4 It will take approximately 24 hours to determine whether blockage is present.

1 A mild sedative is used because the client must be alert enough during the procedure to follow directions. A cardiac catheterization takes approximately 2 hours, not 15 minutes. The client remains on bed rest with the legs extended for 4 to 6 hours after the femoral method of entry. Blockages can be visualized during the procedure.

A 52-yr-old male patient has received a bolus dose and an infusion of alteplase (Activase) for an ST-segment elevation myocardial infarction (STEMI). Which patient assessment would determine the effectiveness of the medication? 1 Presence of chest pain 2 Blood in the urine or stool 3 Tachycardia with hypotension 4 Decreased level of consciousness

1 Alteplase is a fibrinolytic agent that is administered to patients who have had an STEMI. If the medication is effective, the patient's chest pain will resolve because the medication dissolves the thrombus in the coronary artery and results in reperfusion of the myocardium. Bleeding is a major complication of fibrinolytic therapy. Signs of major bleeding include decreased level of consciousness, blood in the urine or stool, and increased heart rate with decreased blood pressure.

An older adult patient with chronic heart failure (HF) and atrial fibrillation asks the nurse why warfarin has been prescribed to continue at home. What is the best response by the nurse? 1 "The medication prevents blood clots from forming in your heart." 2 "The medication dissolves clots that develop in your coronary arteries." 3 "The medication reduces clotting by decreasing serum potassium levels." 4 "The medication increases your heart rate so that clots do not form in your heart."

1 Chronic HF causes enlargement of the chambers of the heart and an altered electrical pathway, especially in the atria. When numerous sites in the atria fire spontaneously and rapidly, atrial fibrillation occurs. Atrial fibrillation promotes thrombus formation within the atria with an increased risk of stroke and requires treatment with cardioversion, antidysrhythmics, and/or anticoagulants. Warfarin is an anticoagulant that interferes with hepatic synthesis of vitamin K-dependent clotting factors.

Which factor should be considered when caring for a woman with suspected coronary artery disease? 1 Fatigue may be the first symptom. 2 Classic signs and symptoms are expected. 3 Increased risk is present before menopause. 4 Women are more likely to develop collateral circulation.

1 Fatigue, rather than pain or shortness of breath, may be the first symptom of impaired cardiac circulation. Women may not exhibit the classic signs and symptoms of ischemia such as chest pain which radiates down the left arm. Neck, throat, or back pain may be symptoms experienced by women. Risk for coronary artery disease increases four times after menopause. Men are more likely to develop collateral circulation.

At a clinic visit, the nurse provides dietary teaching for a patient recently hospitalized with an exacerbation of chronic heart failure. The nurse determines that teaching is successful if the patient makes which statement? 1 "I will limit the amount of milk and cheese in my diet." 2 "I can add salt when cooking foods but not at the table." 3 "I will take an extra diuretic pill when I eat a lot of salt." 4 "I can have unlimited amounts of foods labeled as reduced sodium."

1 Milk products should be limited to 2 cups per day for a 2500-mg sodium-restricted diet. Salt should not be added during food preparation or at the table. Diuretics should be taken as prescribed (usually daily) and not based on sodium intake. Foods labeled as reduced sodium contain at least 25% less sodium than regular.

What is the priority nursing action when caring for a client receiving nitroglycerin for the treatment of angina? 1 Asking the client to sit or stand slowly 2 Monitoring the client's urine output frequently 3 Advising the client to report when experiencing a headache 4 Reporting to the healthcare provider if pain does not subside after 5 minutes

1 Nitroglycerin is a potent antihypertensive and antianginal medication. The nurse should instruct the client to sit and stand slowly after taking the medication to prevent orthostatic hypotension. After ensuring the client's safety, the nurse should monitor the urine output. A headache is a common side effect of nitroglycerin. The client should have a tingling sensation after taking the nitroglycerin, which ensures that the medication is potent.

When caring for a patient in acute septic shock, what should the nurse anticipate? 1 Infusing large amounts of IV fluids 2 Administering osmotic and/or loop diuretics 3 Administering IV diphenhydramine (Benadryl) 4 Assisting with insertion of a ventricular assist device (VAD)

1 Septic shock is characterized by a decreased circulating blood volume. Volume expansion with the administration of IV fluids is the cornerstone of therapy. The administration of diuretics is inappropriate. VADs are useful for cardiogenic shock not septic shock. Diphenhydramine may be used for anaphylactic shock but would not be helpful with septic shock.

A client who is receiving multiple medications for a myocardial infarction complains of severe nausea, and the client's heartbeat is irregular and slow. The nurse determines that these signs and symptoms are toxic effects of what drug? 1 Digoxin 2 Captopril 3 Furosemide 4 Morphine sulfate

1 Signs of digoxin toxicity include cardiac dysrhythmias, anorexia, nausea, vomiting, and visual disturbances. Although nausea and heart block may occur with captopril, these symptoms rarely are seen; drowsiness and central nervous system disturbances are more common. Toxic effects of morphine are slow, deep respirations, stupor, and constricted pupils; nausea is a side effect, not a toxic effect. Toxic effects of furosemide are renal failure, blood dyscrasias, and loss of hearing.

When planning emergent care for a patient with a suspected myocardial infarction (MI), what should the nurse anticipate administrating? 1 Oxygen, nitroglycerin, aspirin, and morphine 2 Aspirin, nitroprusside, dopamine, and oxygen 3 Oxygen, furosemide (Lasix), nitroglycerin, and meperidine 4 Nitroglycerin, lorazepam (Ativan), oxygen, and warfarin (Coumadin)

1 The American Heart Association's guidelines for emergency care of the patient with chest pain include the administration of oxygen, nitroglycerin, aspirin, and morphine. These interventions serve to relieve chest pain, improve oxygenation, decrease myocardial workload, and prevent further platelet aggregation. The other medications may be used later in the patient's treatment.

The patient with chronic heart failure is being discharged from the hospital. What information should the nurse emphasize in the patient's discharge teaching to prevent progression of the disease to acute decompensated heart failure (ADHF)? 1 Take medications as prescribed. 2 Use oxygen when feeling short of breath. 3 Only ask the physician's office questions. 4 Encourage most activity in the morning when rested.

1 The goal for the patient with chronic HF is to avoid exacerbations and hospitalization. Taking the medications as prescribed along with nondrug therapies such as alternating activity with rest will help the patient meet this goal. If the patient needs to use oxygen at home, it will probably be used all the time or with activity to prevent respiratory acidosis. Many HF patients are monitored by a care manager or in a transitional program to assess the patient for medication effectiveness and monitor for patient deterioration and encourage the patient. This nurse manager can be asked questions or can contact the health care provider if there is evidence of worsening HF.

A patient admitted with heart failure is anxious and reports shortness of breath. Which nursing actions would be appropriate to alleviate this patient's anxiety (select all that apply.)? 1 Administer ordered morphine sulfate. 2 Position patient in a semi-Fowler's position. 3 Position patient on left side with head of bed flat. 4 Instruct patient on the use of relaxation techniques. 5 Use a calm, reassuring approach while talking to patient.

1,2,4,5 Morphine sulfate reduces anxiety and may assist in reducing dyspnea. The patient should be positioned in semi-Fowler's position to improve ventilation that will reduce anxiety. Relaxation techniques and a calm reassuring approach will also serve to reduce anxiety.

The patient has heart failure (HF) with an ejection fraction of less than 40%. What core measures should the nurse expect to include in the plan of care for this patient (select all that apply.)? 1 Left ventricular function is documented. 2 Controlling dysrhythmias will eliminate HF. 3 Prescription for digoxin (Lanoxin) at discharge 4 Prescription for angiotensin-converting enzyme inhibitor at discharge 5 Education materials about activity, medications, weight

1,4,5 The Joint Commission has identified these three core measures for heart failure patients. Although controlling dysrhythmias will improve CO and workload, it will not eliminate HF. Prescribing digoxin for all HF patients is no longer done because there are newer effective drugs and digoxin toxicity occurs easily related to electrolyte levels and the therapeutic range must be maintained.

The nurse is caring for a client who had a massive myocardial infarction and developed cardiogenic shock. Which clinical manifestations support these diagnoses? Select all that apply. 1 Rapid pulse 2 Deep respirations 3 Warm, flushed skin 4 Increased blood pressure 5 Decreased urinary output

1,5 The heart rate increases (tachycardia) in an attempt to meet the body's oxygen demands and circulate blood to vital organs; the pulse is weak and thready because of peripheral vasoconstriction. The urinary output decreases because increased catecholamines and activation of the renin-angiotensin-aldosterone system increase fluid reabsorption in the kidneys. The respirations are rapid and shallow, not deep. The skin is cold and clammy because of vasoconstriction caused by the shunting of blood to vital organs. The blood pressure is decreased, not increased, because of continued hypoperfusion and multiorgan failure.

A client who recently had a myocardial infarction is admitted to the cardiac care unit. How can the nurse best determine the effectiveness of the client's ventricular contractions? 1 Observing anxiety levels 2 Monitoring urinary output hourly 3 Evaluating cardiac enzyme results 4 Assessing breath sounds frequently

2 A decreased urinary output reflects a decreased cardiac output; immediate action is indicated if urinary output decreases. Although anxiety may occur, the priority is to monitor urinary output, which reflects cardiac effectiveness. Cardiac enzyme results do not reflect effectiveness of cardiac contractions; they reflect tissue damage. Although the presence of crackles (rales) will indicate pulmonary edema, it will not determine the effectiveness of ventricular contractions.

The nurse is providing teaching to a patient recovering from a myocardial infarction. How should resumption of sexual activity be discussed? 1 Delegated to the primary care provider 2 Discussed along with other physical activities 3 Avoided because it is embarrassing to the patient 4 Accomplished by providing the patient with written material

2 Although some nurses may not feel comfortable discussing sexual activity with patients, it is a necessary component of patient teaching. It is helpful to consider sex as a physical activity and to discuss or explore feelings in this area when other physical activities are discussed. Although providing the patient with written material is appropriate, it should not replace a verbal dialogue that can address the individual patient's questions and concerns.

The home care nurse visits a patient with chronic heart failure. Which clinical manifestations, assessed by the nurse, would indicate acute decompensated heart failure (pulmonary edema)? 1 Fatigue, orthopnea, and dependent edema 2 Severe dyspnea and blood-streaked, frothy sputum 3 Temperature is 100.4oF and pulse is 102 beats/min 4 Respirations 26 breaths/min despite oxygen by nasal cannula

2 Clinical manifestations of pulmonary edema include anxiety, pallor, cyanosis, clammy and cold skin, severe dyspnea, use of accessory muscles of respiration, a respiratory rate greater than 30 breaths/min, orthopnea, wheezing, and coughing with the production of frothy, blood-tinged sputum. Auscultation of the lungs may reveal crackles, wheezes, and rhonchi throughout the lungs. The heart rate is rapid, and blood pressure may be elevated or decreased.

A patient with a recent diagnosis of heart failure has been prescribed furosemide. What outcome does the nurse anticipate will occur that demonstrates medication effectiveness? 1 Promote vasodilation. 2 Reduction of preload. 3 Decrease in afterload. 4 Increase in contractility.

2 Diuretics such as furosemide are used in the treatment of heart failure to mobilize edematous fluid, reduce pulmonary venous pressure, and reduce preload. They do not directly influence afterload, contractility, or vessel tone.

What should the nurse recognize as an indication for the use of dopamine in the care of a patient with heart failure? 1 Acute anxiety 2 Hypotension and tachycardia 3 Peripheral edema and weight gain 4 Paroxysmal nocturnal dyspnea (PND)

2 Dopamine is a β-adrenergic agonist whose inotropic action is used for treatment of severe heart failure accompanied by hemodynamic instability. Such a state may be indicated by tachycardia accompanied by hypotension. PND, anxiety, edema, and weight gain are common signs and symptoms of heart failure, but these do not necessarily warrant the use of dopamine.

The nurse prepares a discharge teaching plan for a 44-yr-old male patient who has recently been diagnosed with coronary artery disease (CAD). Which risk factor should the nurse plan to focus on during the teaching session? 1 Type A personality 2 Elevated serum lipids 3 Family cardiac history 4 Hyperhomocysteinemia

2 Dyslipidemia is one of the four major modifiable risk factors for CAD. The other major modifiable risk factors are hypertension, tobacco use, and physical inactivity. Research findings related to psychologic states (i.e., type A personality) as a risk factor for coronary artery disease have been inconsistent. Family history is a nonmodifiable risk factor. High homocysteine levels have been linked to an increased risk for CAD.

The nurse would recognize which clinical manifestation as suggestive of sepsis? 1 Sudden diuresis unrelated to drug therapy 2 Hyperglycemia in the absence of diabetes 3 Respiratory rate of seven breaths per minute 4 Bradycardia with sudden increase in blood pressure

2 Hyperglycemia in patients with no history of diabetes is a diagnostic criterion for sepsis. Oliguria, not diuresis, typically accompanies sepsis along with tachypnea and tachycardia.

Which patient is at greatest risk for sudden cardiac death (SCD)? 1 A 42-yr-old white woman with hypertension and dyslipidemia 2 A 52-yr-old African American man with left ventricular failure 3 A 62-yr-old obese man with diabetes mellitus and high cholesterol 4 A 72-yr-old Native American woman with a family history of heart disease

2 Patients with left ventricular dysfunction (ejection fraction)

A patient returns after cardiac catheterization. Which nursing care would the registered nurse delegate to the licensed practical nurse? 1 Monitor the electrocardiogram for dysrhythmias 2 Check for bleeding at the catheter insertion site 3 Prepare discharge teaching related to complications 4 Take vital signs and report abnormal values

2 The licensed practical nurse can check for bleeding at the puncture sites. If bleeding is identified, it should be reported to the registered nurse. Vital signs should be delegated to the unlicensed assistive personnel. Preparation of discharge teaching and monitoring for dysrhythmias such as S-T elevation would be registered nurse scope of practice.

The nurse is preparing to administer digoxin to a patient with heart failure. In preparation, laboratory results are reviewed with the following findings: sodium 139 mEq/L, potassium 5.6 mEq/L, chloride 103 mEq/L, and glucose 106 mg/dL. What is the priority action by the nurse? 1 Withhold the daily dose until the following day. 2 Withhold the dose and report the potassium level. 3 Give the digoxin with a salty snack, such as crackers. 4 Give the digoxin with extra fluids to dilute the sodium level.

2 The normal potassium level is 3.5 to 5.0 mEq/L. The patient is hyperkalemic, which makes the patient more prone to digoxin toxicity. For this reason, the nurse should withhold the dose and wait for the potassium level to normalize. The physician may order the digoxin to be given once the potassium level has been treated and decreases to within normal range.

After teaching a patient with chronic stable angina about nitroglycerin, the nurse recognizes the need for further teaching when the patient makes which statement? 1 "I will replace my nitroglycerin supply every 6 months." 2 "I can take up to five tablets every 3 minutes for relief of my chest pain." 3 "I will take acetaminophen (Tylenol) to treat the headache caused by nitroglycerin." 4 "I will take the nitroglycerin 10 minutes before planned activity that usually causes chest pain."

2 The recommended dose of nitroglycerin is one tablet taken sublingually (SL) or one metered spray for symptoms of angina. If symptoms are unchanged or worse after 5 minutes, the patient should be instructed to activate the emergency medical services (EMS) system. If symptoms are improved, repeat the nitroglycerin every 5 minutes for a maximum of three doses and contact EMS if symptoms have not resolved completely.

The nurse is preparing to administer a nitroglycerin patch to a patient. When providing instructions regarding the use of the patch, what should the nurse include in the teaching? 1 Avoid high-potassium foods 2 Avoid drugs to treat erectile dysfunction 3 Avoid over-the-counter H2-receptor blockers 4 Avoid nonsteroidal antiinflammatory drugs (NSAIDS)

2 The use of erectile drugs concurrent with nitrates creates a risk of severe hypotension and possibly death. High-potassium foods, NSAIDs, and H2-receptor blockers do not pose a risk in combination with nitrates.

The nurse is performing an assessment for a patient with fatigue and shortness of breath. Auscultation of the heart reveals the presence of a murmur. What is this assessment finding indicative of? 1 Increased viscosity of the patient's blood 2 Turbulent blood flow across a heart valve 3 Friction between the heart and the myocardium 4 A deficit in heart conductivity that impairs normal contractility

2 Turbulent blood flow across the affected valve results in a murmur. A murmur is not a direct result of variances in blood viscosity, conductivity, or friction between the heart and myocardium.

The nurse would assess a patient with complaints of chest pain for which clinical manifestations associated with a myocardial infarction (MI) (select all that apply.)? 1 Flushing 2 Ashen skin 3 Diaphoresis 4 Nausea and vomiting 5 S3 or S4 heart sounds

2,3,4,5 During the initial phase of an MI, catecholamines are released from the ischemic myocardial cells, causing increased sympathetic nervous system stimulation. This results in the release of glycogen, diaphoresis, and vasoconstriction of peripheral blood vessels. The patient's skin may be ashen, cool, and clammy (not flushed) as a result of this response. Nausea and vomiting may result from reflex stimulation of the vomiting center by severe pain. Ventricular dysfunction resulting from the MI may lead to the presence of the abnormal S3 and S4 heart sounds.

What is the priority assessment by the nurse caring for a patient receiving IV nesiritide to treat heart failure? 1 Urine output 2 Lung sounds 3 Blood pressure 4 Respiratory rate

3 Although all identified assessments are appropriate for a patient receiving IV nesiritide, the priority assessment would be monitoring for hypotension, the main adverse effect of nesiritide.

How can the nurse best describe heart failure to a client? 1 A cardiac condition caused by inadequate circulating blood volume 2 An acute state in which the pulmonary circulation pressure decreases 3 An inability of the heart to pump blood in proportion to metabolic needs 4 A chronic state in which the systolic blood pressure drops below 90 mm Hg

3 As the heart fails, cardiac output decreases; eventually the decrease will reach a level that prevents tissues from receiving adequate oxygen and nutrients. Heart failure is related to an increased, not decreased, circulating blood volume. The condition may be acute or chronic; the pulmonary pressure increases and capillary fluid is forced into the alveoli. The blood pressure usually does not drop.

A 74-yr-old man with a history of prostate cancer and hypertension is admitted to the emergency department with substernal chest pain. Which action will the nurse complete before administering sublingual nitroglycerin? 1 Administer morphine sulfate IV. 2 Auscultate heart and lung sounds. 3 Obtain a 12-lead electrocardiogram (ECG). 4 Assess for coronary artery disease risk factors.

3 If a patient has chest pain, the nurse should institute the following measures: (1) administer supplemental oxygen and position the patient in upright position unless contraindicated, (2) assess vital signs, (3) obtain a 12-lead ECG, (4) provide prompt pain relief first with a nitrate followed by an opioid analgesic if needed, and (5) auscultate heart sounds. Obtaining a 12-lead ECG during chest pain aids in the diagnosis.

A patient's localized infection has become systemic and septic shock is suspected. What medication is expected to treat septic shock refractory to fluids? 1 Insulin infusion 2 Furosemide (Lasix) IV push 3 Norepinephrine administered by titration 4 Administration of nitrates and β-adrenergic blockers

3 If fluid resuscitation using crystalloids is not effective, vasopressor medications such as norepinephrine (Levophed) and dopamine are indicated to restore mean arterial pressure (MAP). Nitrates and β-adrenergic blockers are most often used in the treatment of patients in cardiogenic shock. Furosemide (Lasix) is indicated for patients with fluid volume overload. Insulin infusion may be administered to normalize blood sugar and improve overall outcomes, but it is not considered a medication used to treat shock.

The nurse is caring for a 29-yr-old man who was admitted 1 week ago with multiple rib fractures, pulmonary contusions, and a left femur fracture from a motor vehicle crash. The attending physician states the patient has developed sepsis, and the family members have many questions. Which information should the nurse include when explaining the early stage of sepsis? 1 Antibiotics are not useful when an infection has progressed to sepsis. 2 Weaning the patient away from the ventilator is the top priority in sepsis. 3 Large amounts of IV fluid are required in sepsis to fill dilated blood vessels. 4 The patient has recovered from sepsis if he has warm skin and ruddy cheeks.

3 Patients with sepsis may be normovolemic, but because of acute vasodilation, relative hypovolemia and hypotension occur. Patients in septic shock require large amounts of fluid replacement and may require frequent fluid boluses to maintain circulation. Antibiotics are an important component of therapy for patients with septic shock. They should be started after cultures (e.g., blood, urine) are obtained and within the first hour of septic shock. Oxygenating the tissues is the top priority in sepsis, so efforts to wean septic patients from mechanical ventilation halt until sepsis is resolving. Additional respiratory support may be needed during sepsis. Although cool and clammy skin is present in other early shock states, the patient in early septic shock may feel warm and flushed because of a hyperdynamic state.

The nurse prepares to administer digoxin 0.125 mg to a patient admitted with influenza and a history of chronic heart failure. What should the nurse assess before giving the medication? 1 Prothrombin time 2 Urine specific gravity 3 Serum potassium level 4 Hemoglobin and hematocrit

3 Serum potassium should be monitored because hypokalemia increases the risk for digoxin toxicity. Changes in prothrombin time, urine specific gravity, and hemoglobin or hematocrit would not require holding the digoxin dose.

The nurse is caring for a 72-yr-old man in cardiogenic shock after an acute myocardial infarction. Which clinical manifestations would be most concerning? 1 Restlessness, heart rate of 124 beats/min, and hypoactive bowel sounds 2 Mean arterial pressure of 54 mm Hg; increased jaundice; and cold, clammy skin 3 PaO2 of 38 mm Hg, serum lactate level of 46.5 mcg/dL, and puncture site bleeding 4 Agitation, respiratory rate of 32 breaths/min, and serum creatinine of 2.6 mg/dL

3 Severe hypoxemia, lactic acidosis, and bleeding are clinical manifestations of the irreversible state of shock. Recovery from this stage is not likely because of multiple organ system failure. Restlessness, tachycardia, and hypoactive bowel sounds are clinical manifestations that occur during the compensatory stage of shock. Decreased mean arterial pressure, jaundice, cold and clammy skin, agitation, tachypnea, and increased serum creatinine are clinical manifestations of the progressive stage of shock.

The nurse assesses the right femoral artery puncture site as soon as the patient arrives after having a stent inserted into a coronary artery. The insertion site is not bleeding or discolored. What should the nurse do next to ensure the femoral artery is intact? 1 Palpate the insertion site for induration. 2 Assess peripheral pulses in the right leg. 3 Inspect the patient's right side and back. 4 Compare the color of the left and right legs.

3 The best method to determine that the right femoral artery is intact after inspection of the insertion site is to logroll the patient to inspect the right side and back for retroperitoneal bleeding. The artery can be leaking and blood is drawn into the tissues by gravity. The peripheral pulses, color, and sensation of the right leg will be assessed per agency protocol.

A patient who had bladder surgery 2 days ago develops acute decompensated heart failure (ADHF) with severe dyspnea. Which action by the nurse would be indicated first? 1 Perform a bladder scan to assess for urinary retention. 2 Restrict the patient's oral fluid intake to 500 mL per day. 3 Assist the patient to a sitting position with arms on the overbed table. 4 Instruct the patient to use pursed-lip breathing until the dyspnea subsides.

3 The nurse should place the patient with ADHF in a high Fowler's position with the feet horizontal in the bed or dangling at the bedside. This position helps decrease venous return because of the pooling of blood in the extremities. This position also increases the thoracic capacity, allowing for improved ventilation. Pursed-lip breathing helps with obstructive air trapping but not with acute pulmonary edema. Restricting fluids takes considerable time to have an effect.

Sublingual nitroglycerin tablets are prescribed to control periodic episodes of chest pain in the client with stable angina. Which instruction should the nurse include when teaching the client about sublingual nitroglycerin? 1 Once the tablet is dissolved, spit out the saliva. 2 Take tablets 3 minutes apart up to a maximum of five tablets. 3 Common side effects include headache and low blood pressure. 4 Once opened, the tablets should be refrigerated to prevent deterioration.

3 The primary side effects of nitroglycerin are headache and hypotension. It is not necessary to spit out saliva into which nitroglycerin has dissolved. For pain that is not relieved, additional tablets may be taken every 5 minutes up to a total of three tablets. It should be stored at room temperature.

The UAP is taking orthostatic vital signs. In the supine position, the blood pressure (BP) is 130/80 mm Hg, and the heart rate (HR) is 80 beats/min. In the sitting position, the BP is 140/80, and the HR is 90 beats/min. Which action should the nurse instruct the UAP to take next? 1 Repeat BP and HR in this position. 2 Record the BP and HR measurements. 3 Take BP and HR with patient standing. 4 Return the patient to the supine position

3 The vital signs taken do not reflect orthostatic changes, so the UAP will continue with the measurements while the patient is standing. There is no need to repeat or delay the readings. The patient does not need to return to the supine positon. When assessing for orthostatic changes, the UAP will take the BP and pulse in the supine position, then place the patient in a sitting position for 1 to 2 minutes and repeat the readings, and then reposition to the standing position for 1 to 2 minutes and repeat the readings. Results consistent with orthostatic changes would have a decrease of 20 mm Hg or more in systolic BP, a decrease of 10 mm Hg or more in diastolic BP, and/or an increase in HR of greater than or equal to 20 beats/min with position changes.

In caring for the patient with angina, the patient said, "While I was having a bowel movement, I started having the worst chest pain ever, like before I was admitted. I called for a nurse, then the pain went away." What further assessment data should the nurse obtain from the patient? 1 "What precipitated the pain?" 2 "Has the pain changed this time?" 3 "In what areas did you feel this pain?" 4 "What is your pain level on a 0 to 10 scale?"

3 Using PQRST, the assessment data not volunteered by the patient is the radiation of pain, the area the patient felt the pain, and if it radiated. The precipitating event was going to the bathroom and having a bowel movement. The quality of the pain was "like before I was admitted," although a more specific description may be helpful. Severity of the pain was the "worst chest pain ever," although an actual number may be needed. Timing is supplied by the patient describing when the pain occurred and that he had previously had this pain.

A patient experienced sudden cardiac death (SCD) and survived. Which preventive treatment should the nurse expect to be implemented? 1 External pacemaker 2 An electrophysiologic study (EPS) 3 Medications to prevent dysrhythmias 4 Implantable cardioverter-defibrillator (ICD)

4 An ICD is the most common approach to preventing recurrence of SCD. An external pacemaker may be used in the hospital but will not be used for the patient living daily life at home. An EPS may be done to determine if a recurrence is likely and determine the most effective medication treatment. Medications to prevent dysrhythmias are used but are not the best prevention of SCD.

After having a myocardial infarction (MI), the nurse notes the patient has jugular venous distention, gained weight, developed peripheral edema, and has a heart rate of 108 beats/min. What should the nurse suspect is happening? 1 ADHF 2 Chronic HF 3 Left-sided HF 4 Right-sided HF

4 An MI is a primary cause of heart failure. The jugular venous distention, weight gain, peripheral edema, and increased heart rate are manifestations of right-sided heart failure.

The nurse is administering a dose of digoxin to a patient with heart failure (HF). The nurse would become concerned with the possibility of digitalis toxicity if the patient reported which symptom? 1 Muscle aches 2 Constipation 3 Pounding headache 4 Anorexia and nausea

4 Anorexia, nausea, vomiting, blurred or yellow vision, and cardiac dysrhythmias are all signs of digitalis toxicity. The nurse would become concerned and notify the health care provider if the patient exhibited any of these symptoms.

A patient is scheduled for a heart transplant. Beyond the first year after a heart transplant, the nurse knows that what is a major cause of death? 1 Infection 2 Acute rejection 3 Immunosuppression 4 Cardiac vasculopathy

4 Beyond the first year after a heart transplant, malignancy (especially lymphoma) and cardiac vasculopathy (accelerated coronary artery disease) are the major causes of death. During the first year after transplant, infection and acute rejection are the major causes of death. Immunosuppressive therapy will be used for posttransplant management to prevent rejection and increases the patient's risk of an infection.

When evaluating a patient's knowledge regarding a low-sodium, low-fat cardiac diet, the nurse recognizes additional teaching is needed when the patient selects which food? 1 Baked flounder 2 Angel food cake 3 Baked potato with margarine 4 Canned chicken noodle soup

4 Canned soups are very high in sodium content. Patients need to be taught to read food labels for sodium and fat content.

A patient admitted to the emergency department 24 hours ago with complaints of chest pain was diagnosed with a ST-segment-elevation myocardial infarction (STEMI). What complication of myocardial infarction should the nurse anticipate? 1 Unstable angina 2 Cardiac tamponade 3 Sudden cardiac death 4 Cardiac dysrhythmias

4 Dysrhythmias are present in 80% to 90% of patients after myocardial infarction (MI). Unstable angina is considered a precursor to MI rather than a complication. Cardiac tamponade is a rare event, and sudden cardiac death is defined as an unexpected death from cardiac causes. Cardiac dysfunction in the period following an MI would not be characterized as sudden cardiac death.

A male patient who has coronary artery disease (CAD) has serum lipid values of low-density lipoprotein (LDL) cholesterol of 98 mg/dL and high-density lipoprotein (HDL) cholesterol of 47 mg/dL. What should the nurse include in patient teaching? 1 Consume a diet low in fats. 2 Reduce total caloric intake. 3 Increase intake of olive oil. 4 The lipid levels are normal.

4 For men, the recommended LDL is less than 100 mg/dL, and the recommended level for HDL is greater than 40mg/dL. His normal lipid levels should be included in the patient teaching and encourage him to continue taking care of himself. Assessing his need for teaching related to diet should also be done.

A female patient who has type 1 diabetes mellitus has chronic stable angina that is controlled with rest. She states that over the past few months, she has required increasing amounts of insulin. What goal should the nurse use to plan care that should help prevent cardiovascular disease progression? 1 Exercise almost every day. 2 Avoid saturated fat intake. 3 Limit calories to daily limit. 4 Keep Hgb A1C (A1C) less than 7%.

4 If the Hgb A1C (A1C) is kept below 7%, this means that the patient has had good control of her blood glucose over the past 3 months. The patient indicates that increasing amounts of insulin are being required to control her blood glucose. This patient may not be adhering to the dietary guidelines or therapeutic regimen, so teaching about how to maintain diet, exercise, and medications to maintain stable blood glucose levels will be needed to achieve this goal.

The patient is being dismissed from the hospital after acute coronary syndrome and will be attending rehabilitation. What information would be taught in the early recovery phase of rehabilitation? 1 Therapeutic lifestyle changes should become lifelong habits. 2 Physical activity is always started in the hospital and continued at home. 3 Attention will focus on management of chest pain, anxiety, dysrhythmias, and other complications. 4 Activity level is gradually increased under cardiac rehabilitation team supervision and with electrocardiographic (ECG) monitoring.

4 In the early recovery phase after the patient is dismissed from the hospital, the activity level is gradually increased under supervision and with ECG monitoring. The late recovery phase includes therapeutic lifestyle changes that become lifelong habits. In the first phase of recovery, activity is dependent on the severity of the angina or myocardial infarction, and attention is focused on the management of chest pain, anxiety, dysrhythmias, and other complications. With early recovery phase, the cardiac rehabilitation team may suggest that physical activity be initiated at home, but this is not always done.

For which problem is percutaneous coronary intervention (PCI) most clearly indicated? 1 Chronic stable angina 2 Left-sided heart failure 3 Coronary artery disease 4 Acute myocardial infarction

4 PCI is indicated to restore coronary perfusion in cases of myocardial infarction. Chronic stable angina and coronary artery disease are normally treated with more conservative measures initially. PCI is not relevant to the pathophysiology of heart failure.

After coronary artery bypass graft surgery a patient has postoperative bleeding that requires returning to surgery for repair. During surgery, the patient has a myocardial infarction (MI). After restoring the patient's body temperature to normal, which patient parameter is the most important for planning nursing care? 1 Cardiac index (CI) of 5 L/min/m2 2 Central venous pressure of 8 mm Hg 3 Mean arterial pressure (MAP) of 86 mm Hg 4 Pulmonary artery pressure (PAP) of 28/14 mm Hg

4 Pulmonary hypertension as indicated by an elevated PAP indicates impaired forward flow of blood because of left ventricular dysfunction or hypoxemia. Both can be a result of the MI. The CI, CVP, and MAP readings are normal.

A patient with a long-standing history of heart failure recently qualified for hospice care. What measure should the nurse now prioritize when providing care for this patient? 1 Taper the patient off his current medications. 2 Continue education for the patient and his family. 3 Pursue experimental therapies or surgical options. 4 Choose interventions to promote comfort and prevent suffering.

4 The central focus of hospice care is the promotion of comfort and the prevention of suffering. Patient education should continue, but providing comfort is paramount. Medications should be continued unless they are not tolerated. Experimental therapies and surgeries are not commonly used in the care of hospice patients.

Which is the priority nursing action to decrease the risk for a client developing a hospital-acquired infection? 1 Using droplet precautions 2 Using contact precautions 3 Using airborne precautions 4 Using standard precautions

4 The use of standard precautions, which includes hand hygiene, is the priority action to decrease a client's risk for developing a hospital-acquired infection. While transmission-based precautions such as droplet, contact, and airborne precautions may be necessary to prevent the transmission of communicable infection to other clients, these are not nursing actions to decrease a client's risk for developing a hospital-acquired infection.

What intervention is included in the nursing care plan for a 4-month-old infant with tetralogy of Fallot and heart failure? 1 Providing small, frequent feedings 2 Positioning the child flat on the back 3 Encouraging nutritional fluids often 4 Measuring the head circumference daily

1 Small, frequent feedings with adequate rest periods in between may improve the infant's intake at each feeding; infants with tetralogy of Fallot become extremely fatigued while suckling. Positioning the child with the head elevated, not flat on the back, facilitates respiration; an infant cardiac seat, similar to a car seat, helps maintain the child in the semi-Fowler position. As a means of reducing the cardiac workload, excessive fluids usually are not offered, and fluids may even be restricted. The head circumference is not an important assessment for infants with congenital heart disease; daily head measurements should be taken for infants with hydrocephaly.

Which individuals would the nurse identify as having the highest risk for coronary artery disease (CAD)? 1 A 45-yr-old depressed man with a high-stress job 2 A 60-yr-old man with below normal homocysteine levels 3 A 54-yr-old woman vegetarian with elevated high-density lipoprotein (HDL) levels 4 A 62-yr-old woman who has a sedentary lifestyle and body mass index (BMI) of 23 kg/m2

1 The 45-yr-old depressed man with a high-stress job is at the highest risk for CAD. Studies demonstrate that depression and stressful states can contribute to the development of CAD. Elevated HDL levels and low homocysteine levels actually help to prevent CAD. Although a sedentary lifestyle is a risk factor, a BMI of 23 kg/m2 depicts normal weight, and thus the patient with two risk factors is at greatest risk for developing CAD.

When providing nutritional counseling for patients at risk for coronary artery disease (CAD), which foods would the nurse encourage patients to include in their diet (select all that apply.)? 1 Tofu 2 Walnuts 3 Tuna fish 4 Whole milk 5 Orange juice

1,2,3 Tuna fish, tofu, and walnuts are all rich in omega-3 fatty acids, which have been shown to reduce the risks associated with CAD when consumed regularly.

A nurse is caring for a client with a diagnosis of right ventricular heart failure. The nurse expects what assessment findings associated with right-sided heart failure? Select all that apply. 1 Dependent edema 2 Swollen hands and fingers 3 Collapsed neck veins 4 Right upper quadrant discomfort 5 Oliguria

1,2,4 With right-sided heart failure, signs of systemic congestion occur as the right ventricle fails; key features include dependent edema and swollen hands and fingers. Upper right quadrant discomfort is expected with right ventricular failure because venous congestion in the systemic circulation results in hepatomegaly. Jugular venous collapse and oliguria are key features of left-sided heart failure. Left-sided heart failure is associated with decreased cardiac output.

What are the clinical manifestations of myocardial infarction in women? Select all that apply. 1 Anoxia 2 Indigestion 3 Unusual fatigue 4 Sleep disturbances 5 Tightness of the chest

2,3,4 Indigestion, unusual fatigue, and sleep disturbances are clinical manifestations of myocardial infarction in women. Anoxia and tightness of the chest are clinical manifestations of angina pectoris, not myocardial infarction.

The nurse is planning to teach a client with heart failure about the signs and symptoms of cardiac decompensation. What clinical manifestations should the nurse include? Select all that apply. 1 Weight loss 2 Extreme fatigue 3 Coughing at night 4 Excessive urination 5 Difficulty breathing

2,3,5 Fatigue is caused by a lack of adequate oxygenation of body cells caused by a decreased cardiac output. As the cardiac output decreases, pulmonary congestion increases, resulting in pulmonary edema; coughing, especially when lying down, and blood-tinged sputum occur. Auscultation reveals crackles and rhonchi. Dyspnea is associated with pulmonary edema that occurs as cardiac output decreases and pulmonary congestion increases. Weight gain, not loss, occurs as fluid is retained by the kidneys. Fluid retention, not diuresis, occurs because of decreased circulation to the kidneys, resulting from decreased cardiac output.

A client has left ventricular heart failure. For which clinical indicators should the nurse assess the client? Select all that apply. 1 Ascites 2 Crackles 3 Peripheral edema 4 Dyspnea on exertion 5 Jugular vein distention

2,4 Pressure in the pulmonic circulation increases when the left ventricle fails; fluid in the alveoli impairs gas exchange, which causes dyspnea on exertion. Ascites, a sign of right ventricular failure, results from an increased hydrostatic pressure in the systemic circulation; fluid moves out of the intravascular compartment into the abdominal cavity. Peripheral edema, a sign of right ventricular failure, results from an increased hydrostatic pressure in the systemic circulation. Fluid moves out of the intravascular compartment into the interstitial compartment. Jugular vein distention, a sign of right ventricular failure, results from hypervolemia.

A client with a history of angina is scheduled for a cardiac catheterization. Catheter entry will be through the femoral artery. What should the nurse tell the client to expect? 1 Remain fully alert during the procedure 2 Ambulate shortly after the procedure 3 Experience a feeling of warmth during the procedure 4 Be placed in a semi-Fowler position for 12 hours after the procedure

3 A warm flushing sensation that lasts approximately 30 seconds will occur when the contrast medium is injected. Medication is given for mild sedation; clients are drowsy but awake enough to follow instructions. The supine position will be maintained for 4 to 6 hours after the procedure; walking may dislodge clots at the catheter insertion site, resulting in bleeding. The semi-Fowler position flexes the legs, which may result in bleeding at the femoral insertion site, and should be avoided.

A client is admitted to the cardiac care unit with a myocardial infarction. The cardiac monitor reveals several runs of ventricular tachycardia. The nurse anticipates that the client will be receiving a prescription for which drug? 1 Atropine 2 Epinephrine 3 Amiodarone 4 Sodium bicarbonate

3 Amiodarone suppresses ventricular activity; therefore, it is used for treatment of premature ventricular complexes (PVCs). It works directly on the heart tissue and slows the nerve impulses in the heart. Atropine blocks vagal stimulation; it increases the heart rate and is used for bradycardia, not PVCs. Epinephrine increases myocardial contractility and heart rate; therefore, it is contraindicated in the treatment of PVCs. Sodium bicarbonate increases the serum pH level; therefore, it combats metabolic acidosis.

An older male patient visits his primary care provider because of burning on urination and production of foul-smelling urine. What contributing factor should the health care provider consider? 1 High-purine diet 2 Sedentary lifestyle 3 Benign prostatic hyperplasia (BPH) 4 Recent use of broad-spectrum antibiotics

3 BPH causes urinary stasis, which is a predisposing factor for UTIs. A sedentary lifestyle and recent antibiotic use are unlikely to contribute to UTIs, but a diet high in purines is associated with renal calculi.

Postoperative care of a patient undergoing coronary artery bypass graft (CABG) surgery includes monitoring for which common complication? 1 Dehydration 2 Paralytic ileus 3 Atrial dysrhythmias 4 Acute respiratory distress syndrome

3 Postoperative dysrhythmias, specifically atrial dysrhythmias, are common in the first 3 days after CABG surgery. Although the other complications could occur, they are not common complications.

The nurse is teaching a women's group about prevention of hypertension. What information should be included in the teaching for all the women (select all that apply.)? 1 Lose weight. 2 Limit nuts and seeds. 3 Limit sodium and fat intake. 4 Increase fruits and vegetables. 5 Exercise 30 minutes most days.

3,4,5 Primary prevention of hypertension is to make lifestyle modifications that prevent or delay the increase in BP. Along with exercise for 30 minutes on most days, the DASH eating plan is a healthy way to lower BP by limiting sodium and fat intake, increasing fruits and vegetables, and increasing nutrients that are associated with lowering BP. Nuts and seeds and dried beans are used for protein intake. Weight loss may or may not be necessary for the individual.

A client reports urinary frequency and burning. To determine whether there is tenderness that indicates the presence of an ascending urinary tract infection, the nurse should palpate which area? 1 Tail of Spence 2 Suprapubic area 3 McBurney point 4 Costovertebral angle

4 The costovertebral angle [1] [2] [3] (the angle formed by the lateral and downward curve of the lowest rib and the vertebral column of the spine itself) is percussed to determine whether there is tenderness in the area over the kidney; this can be a sign of glomerulonephritis or severe upper urinary tract infection. The tail of Spence extends from the upper outer quadrant of the breast to the axillary area; this is the most common site for tumors associated with cancer of the breast. The suprapubic area is above the symphysis pubis; it is palpated and percussed to assess for bladder distention. McBurney's point is 1 to 2 inches (2.5 to 5 cm) above the anterosuperior spine of the ileum on a line between the ileum and umbilicus; external pressure produces tenderness with acute appendicitis, not a kidney infection.

A healthcare provider prescribes tissue plasminogen activator (t-PA) to be administered intravenously over 1 hour for a client experiencing a myocardial infarction. What is the nurse's priority assessment that is specific to this medication's effect? 1 Respiratory rate 2 Peripheral pulses 3 Level of consciousness 4 Intravenous insertion site

4 The most common adverse effect of a tissue plasminogen activator is bleeding because of the thrombolytic action of the drug. Sites of invasive procedures, such as IV sites, have an increased tendency to bleed. Although respiratory rate, peripheral pulses, and level of consciousness are important for any client with a decreased cardiac output, they are not specific to the administration of a tissue plasminogen activator.

An asymptomatic patient with acute decompensated heart failure (ADHF) suddenly becomes dyspneic. Before dangling the patient on the bedside, what should the nurse assess first? 1 Urine output 2 Heart rhythm 3 Breath sounds 4 Blood pressure

4 The nurse should evaluate the blood pressure before dangling the patient on the bedside because the blood pressure can decrease as blood pools in the periphery and preload decreases. If the patient's blood pressure is low or marginal, the nurse should put the patient in the semi-Fowler's position and use other measures to improve gas exchange.

What laboratory finding is consistent with a medical diagnosis of cardiogenic shock? 1 Decreased liver enzymes 2 Increased white blood cells 3 Decreased red blood cells, hemoglobin, and hematocrit 4 Increased blood urea nitrogen (BUN) and serum creatinine (Cr) levels

4 The renal hypoperfusion that accompanies cardiogenic shock results in increased BUN and creatinine levels. Impaired perfusion of the liver results in increased liver enzymes, but white blood cell levels do not typically increase in cardiogenic shock. Red blood cell indices are typically normal because of relative hypovolemia.

In palpating the patient's pedal pulses, the nurse determines the pulses are absent. What factor could contribute to this result? 1 Atherosclerosis 2 Hyperthyroidism 3 Arteriovenous fistula 4 Cardiac dysrhythmias

1 Atherosclerosis can cause an absent peripheral pulse. The feet would also be cool and may be discolored. Hyperthyroidism causes a bounding pulse. Arteriovenous fistula gives a thrill or vibration to the vessel, although this would not be in the foot. Cardiac dysrhythmias cause an irregular pulse rhythm.

The nurse should suspect that a client who had a recent myocardial infarction is experiencing denial when the client does what? 1 Attempts to minimize the illness 2 Lacks an emotional response to the illness 3 Refuses to discuss the condition with the client's spouse 4 Expresses displeasure with the prescribed activity program

1 Attempts to minimize the illness is a classic sign of denial; by reducing the importance or extent of the problem, the individual is able to cope. Not acknowledging that it is really a problem is a form of denial. Lacking an emotional response to the illness indicates repression of affect rather than denial. Failure to communicate is insufficient evidence to diagnose denial; the marital relationship may be strained, or the client may be worried about upsetting the spouse. Expressing displeasure with the activity program usually indicates displacement of anger, not denial.

The nurse is admitting a patient who is scheduled to undergo a cardiac catheterization. What allergy information is most important for the nurse to assess and document before this procedure? 1 Iron 2 Iodine 3 Aspirin 4 Penicillin

2 The physician will usually use an iodine-based contrast to perform this procedure. Therefore it is imperative to know whether or not the patient is allergic to iodine or shellfish. Knowledge of allergies to iron, aspirin, or penicillin will be secondary.

A client is admitted with cellulitis of the left leg and a temperature of 103° F (39.4° C). The primary healthcare provider prescribes intravenous (IV) antibiotics. Which action is the priority before administering the antibiotics? 1 Determine the client's allergies. 2 Apply a warm, moist dressing over the cellulitis. 3 Measure the amount of swelling in the client's left leg. 4 Obtain the results of the culture and sensitivity tests.

1 Allergies are important. Drug hypersensitivity and anaphylaxis are most common with antimicrobial agents. Applying a warm, moist dressing over the area is a dependent function; it is not crucial to starting antibiotic therapy. Measuring the amount of swelling in the client's leg is an important assessment, but it is not crucial to starting antibiotic therapy. Withholding treatment until culture results are available may extend the infection.

A client admitted to the hospital for chest pain is diagnosed with stable angina. Which information should the nurse include in the teaching session? 1 It is relieved by rest. 2 It is precipitated by light activity. 3 It is described as sharp or knifelike. 4 It is unaffected by the administration of vasodilators.

1 Anginal pain commonly is relieved by immediate rest because rest decreases the cardiac workload and oxygen need. Angina usually is precipitated by exertion, emotion, or a heavy meal. Anginal pain usually is described as tightness, indigestion, or heaviness. Nitroglycerin, a vasodilator and a standard treatment for angina, dilates coronary arteries, which increases oxygen to the myocardium, decreasing pain.

The nurse is providing care for a patient who has decreased cardiac output related to heart failure. What should the nurse recognize about cardiac output? 1 It is calculated by multiplying the patient's stroke volume by the heart rate. 2 It is the average amount of blood ejected during one complete cardiac cycle. 3 It is determined by measuring the electrical activity of the heart and the patient's heart rate. 4 It is the patient's average resting heart rate multiplied by the patient's mean arterial blood pressure.

1 Cardiac output is determined by multiplying the patient's stroke volume by heart rate, thus identifying how much blood is pumped by the heart over a 1-minute period. Electrical activity of the heart and blood pressure are not direct components of cardiac output.

A nurse is assessing the urine of a client with a urinary tract infection. For which characteristic should the nurse assess each specimen of urine? 1 Clarity 2 Viscosity 3 Glucose level 4 Specific gravity

1 Cloudy urine usually indicates drainage associated with infection. Viscosity is a characteristic that is not measurable in urine. Urinary glucose levels are not affected by urinary tract infections. Specific gravity yields information related to fluid balance.

The nurse assesses bilateral +4 peripheral edema while assessing a client with heart failure and peripheral vascular disease. What is the pathophysiological reason for the excessive edema? 1 Shift of fluid into the interstitial spaces 2 Weakening of the cell wall 3 Increased intravascular compliance 4 Increased intracellular fluid volume

1 Edema is defined as the accumulation of fluid in the interstitial spaces. When the heart is unable to maintain adequate blood flow throughout the circulatory system, the excess fluid pressure within the blood vessels can cause shifts into the interstitial spaces. Weakening of the cell wall may cause leakage of fluid, but this is not the pathologic reason related to heart failure. Increased intravascular compliance would prevent fluid from shifting into the tissue. Intracellular volume is maintained within the cell and not in the tissue.

The nurse is preparing discharge instructions for a client who was prescribed enalapril for treatment of hypertension. Which instruction is appropriate for the nurse to include in the client's teaching? 1 Do not change to a standing position suddenly. 2 Lightheadedness is a common adverse effect that need not be reported. 3 The medication may cause a sore throat for the first few days. 4 Schedule blood tests weekly for the first 2 months.

1 Enalapril is classified as an angiotensin-converting enzyme (ACE) inhibitor. It is used to treat hypertension and congestive heart failure. It can also be used to treat a disorder of the ventricles. Angiotensin is a chemical that causes the arteries to become narrow. ACE inhibitors help the body produce less angiotensin, which helps the blood vessels relax and open up, which, in turn, lowers blood pressure. Clients should be advised to change positions slowly to minimize orthostatic hypotension. A healthcare provider should be notified immediately if the client is experiencing lightheadedness or feeling like he or she is about to faint, as this is a serious side effect. This medication does not cause a sore throat the first few days of treatment. Presently, there are no guidelines that suggest blood tests are required weekly for the first 2 months.

A 5-month-old infant is admitted with a diagnosis of respiratory syncytial virus (RSV) infection. The infant's condition suddenly deteriorates, and a dose of epinephrine is prescribed to relieve bronchospasm. For what side effect of the medication should the nurse assess the infant? 1 Tachycardia 2 Hypotension 3 Respiratory arrest 4 Central nervous system depression

1 Epinephrine stimulates beta- and alpha-receptors; its actions include increasing heart rate and blood pressure and inducing bronchodilation. Increased blood pressure, not hypotension, is a potential side effect. Epinephrine relieves respiratory problems; it does not cause respiratory arrest. Epinephrine stimulates, not depresses, the central nervous system.

An infant with bronchiolitis caused by respiratory syncytial virus (RSV) is admitted to the pediatric unit. What does the nurse expect the prescribed treatment to include? 1 Humidified cool air and adequate hydration 2 Postural drainage and oxygen by hood 3 Bronchodilators and cough suppressants 4 Corticosteroids and broad-spectrum antibiotics

1 Humidified cool air and hydration are essential to facilitating improvement in the child's physical status. Postural drainage is not effective with this disorder; oxygen is used only if the infant has severe dyspnea and hypoxia. Bronchodilators are not used, because the bronchial tree is not in spasm; cough suppressants are ineffective. Corticosteroids are ineffective; antibiotics are also ineffective, because the causative agent is viral.

The nurse teaches a patient with hypertension that uncontrolled hypertension may damage organs in the body primarily by which mechanism? 1 Hypertension promotes atherosclerosis and damage to the walls of the arteries. 2 Hypertension causes direct pressure on organs, resulting in necrosis and replacement of cells with scar tissue. 3 Hypertension causes thickening of the capillary membranes, leading to hypoxia of organ systems. 4 Hypertension increases blood viscosity, which contributes to intravascular coagulation and tissue necrosis distal to occlusions.

1 Hypertension is a major risk factor for the development of atherosclerosis by mechanisms not yet fully known. However, when atherosclerosis develops, it damages the walls of arteries and reduces circulation to target organs and tissues.

To help prevent a cycle of recurring urinary tract infections in a female client, which instruction should the nurse share? 1 "Urinate as soon as possible after intercourse." 2 "Increase your daily intake of citrus juice." 3 "Douche regularly with alkaline agents." 4 "Take bubble baths regularly."

1 Intercourse may cause urethral inflammation, increasing the risk of infection; voiding clears the urinary meatus and urethra of microorganisms. Most fruit juices, with the exception of cranberry juice, cause alkaline urine, which promotes bacterial growth. Douching is no longer recommended because it alters the vaginal flora. Bubble baths can promote urinary tract infections.

A client with heart failure has anxiety. Which effect of anxiety makes it particularly important for the nurse to reduce the anxiety of this client? 1 Increases the cardiac workload 2 Interferes with usual respirations 3 Produces an elevation in temperature 4 Decreases the amount of oxygen used

1 Irritability and restlessness associated with anxiety increase the metabolic rate, heart rate, and blood pressure; these complicate heart failure. Anxiety does not directly interfere with respirations; an increase in cardiac workload will increase respirations. Anxiety alone usually does not elevate the body temperature. Anxiety can cause an increase in the amount of oxygen used and leads to an increased respiratory rate.

.A client is taking furosemide and digoxin for heart failure. Why does the nurse advise the client to drink a glass of orange juice every day? 1 Maintaining potassium levels 2 Preventing increased sodium levels 3 Limiting the drugs' synergistic effects 4 Correcting the associated dehydration

1 Orange juice is an excellent source of potassium. Furosemide promotes excretion of potassium, which can result in hypokalemia. Digoxin toxicity can occur in the presence of hypokalemia. Neither drug increases sodium levels. Digoxin does not potentiate the action of furosemide; therefore, the client should not experience dehydration. Orange juice will not prevent an interaction between digoxin and furosemide.

A nurse is caring for a client who has had multiple myocardial infarctions and has now developed cardiogenic shock. Which clinical manifestation supports this diagnosis? 1 Cold, clammy skin 2 Slow, bounding pulse 3 Increased blood pressure 4 Hyperactive bowel sounds

1 The action of the sympathetic nervous system causes vasoconstriction, and as cellular and peripheral hypoperfusion progresses, the skin becomes cold, clammy, cyanotic, or mottled. The heart rate increases in an attempt to meet the body's oxygen demands and circulate blood to vital organs; it has a low volume (weak, thready) because of peripheral vasoconstriction. The blood pressure decreases because of continued hypoperfusion and multiorgan failure. Bowel sounds are hypoactive or absent, not hyperactive.

Nitroglycerin sublingual tablets are prescribed for a client with the diagnosis of angina. The client asks the nurse how long it should take for the chest pain to subside after nitroglycerin is taken. What should the nurse tell the client? 1 1 to 3 minutes 2 4 to 5 seconds 3 30 to 45 seconds 4 20 to 45 minutes

1 The onset of action of sublingual nitroglycerin tablets is rapid (1 to 3 minutes); duration of action is 30 to 60 minutes. If nitroglycerin is administered intravenously, the onset of action is immediate, and the duration is 3 to 5 minutes. It takes longer than 30 to 45 seconds for sublingual nitroglycerin tablets to have a therapeutic effect. Sustained-release nitroglycerin tablets start to act in 20 to 45 minutes, and the duration of action is 3 to 8 hours.

When caring for elderly patients with hypertension, which information should the nurse consider when planning care (select all that apply.)? 1 Systolic blood pressure increases with aging. 2 Blood pressures should be maintained near 120/80 mm Hg. 3 White coat syndrome is prevalent in elderly patients. 4 Volume depletion contributes to orthostatic hypotension. 4 Blood pressure drops 1 hour postprandially in many older patients. 5 Older patients will require higher doses of antihypertensive medications.

1,3,4,5 Systolic blood pressure increases with age and patients older than age 60 years should be maintained below 150/90 mm Hg. Older patients have significantly higher blood pressure readings when taken by health care providers (white coat syndrome). Older patients experience orthostatic hypotension related to dehydration, reduced compensatory mechanisms, and medications. One hour after eating, many older patients experience a drop in blood pressure. Lower doses of medications may be needed to control blood pressures in older adults related to decreased absorption rates and excretion ability.

After reviewing the urinalysis reports of a client with a renal disorder, the nurse concludes that the client may have a urinary tract infection. Which urinary laboratory findings enabled the nurse to make this conclusion? Select all that apply. 1 pH: 8.5 2 Specific gravity: 1.010 3 Red blood cells: 3/hpf 4 Osmolality: 1500 mOsm/kg (1500 mmol/kg) 5 White blood cells: 6/hpf

1,5 The client may have a urinary tract infection, as the urinalysis reports show the presence of pH as 8.5 and white blood cells as 6/hpf in the urine. A pH above 8.0 indicates a urinary tract infection; client's is 8.5. The normal level of white blood cells (WBC) in urine should be less than 5/hpf; therefore, the WBC level of 6/hpf indicates urinary tract infection. The specific gravity of 1.010 indicates a normal finding. The normal level of red blood cells (RBC) is less than 4/hpf; therefore, the RBC levels of 3/hpf indicates normal finding. Osmolality of 1500 mOsm/kg (1500 mmol/kg) indicates tubular dysfunction.

A client who develops heart failure has a serum potassium level of 2.3 mEq/L (2.3 mmol/L). Digoxin and potassium chloride are prescribed. What action should the nurse take? 1 Double the dose of potassium chloride and administer it with the prescribed digoxin. 2 Hold the dose of digoxin, administer the potassium chloride, and call the primary healthcare provider immediately. 3 Give the digoxin and potassium chloride as prescribed and report the laboratory results to the primary healthcare provider. 4 Administer the prescribed digoxin and potassium chloride with a glass of orange juice and continue to monitor the client.

2 A low potassium level with the administration of digoxin can cause digitalis toxicity, resulting in life-threatening dysrhythmias. Doubling the dose of potassium chloride and administering it with the prescribed digoxin has the potential of causing digitalis toxicity. In addition, changing the dose of a medication is not within the legal role of the nurse and requires a primary healthcare provider's prescription. Giving the digoxin and potassium chloride as prescribed and reporting the laboratory results to the primary healthcare provider has the potential of causing digitalis toxicity, especially when the potassium level is less than 3 mEq/L (3 mmol/L). Administering the prescribed digoxin and potassium chloride with a glass of orange juice and continuing to monitor the client has the potential of causing digitalis toxicity. One glass of orange juice and one dose of potassium chloride will not change the potassium level significantly.

The nurse is performing an assessment for a patient undergoing radiation treatment for breast cancer. What position should the nurse place the patient to best auscultate for signs of acute pericarditis? 1 Supine without a pillow 2 Sitting and leaning forward 3 Left lateral side-lying position 4 Head of bed at a 45-degree angle

2 A pericardial friction rub indicates pericarditis. To auscultate a pericardial friction rub, the patient should be sitting and leaning forward. The nurse will hear the pericardial friction rub at the end of expiration.

The nurse is caring for a patient with hypertension who is scheduled to receive a dose of metoprolol (Lopressor). The nurse should withhold the dose and consult the prescribing physician for which vital sign taken just before administration? 1 O2 saturation 93% 2 Pulse 48 beats/min 3 Respirations 24 breaths/min 4 Blood pressure 118/74 mm Hg

2 Because metoprolol is a β1-adrenergic blocking agent, it can cause hypotension and bradycardia as adverse effects. The nurse should withhold the dose and consult with the health care provider for parameters regarding pulse rate limits.

The nurse admits a 73-yr-old male patient with dementia for treatment of uncontrolled hypertension. The nurse will closely monitor for hypokalemia if the patient receives which medication? 1 Clonidine (Catapres) 2 Bumetanide (Bumex) 3 Amiloride (Midamor) 4 Spironolactone (Aldactone)

2 Bumetanide is a loop diuretic. Hypokalemia is a common adverse effect of this medication. Amiloride is a potassium-sparing diuretic. Spironolactone is an aldosterone-receptor blocker. Hyperkalemia is an adverse effect of both amiloride and spironolactone. Clonidine is a central-acting α-adrenergic antagonist and does not cause electrolyte abnormalities.

Within 4 to 6 hours after a client has a myocardial infarction, the nurse expects which blood level to increase? 1 Lactate dehydrogenase (LDH-1) 2 Creatine kinase-MB band (CK-MB) 3 Erythrocyte sedimentation rate (ESR) 4 Serum aspartate aminotransferase (AST)

2 CK-MB is an isoenzyme of creatine phosphokinase (CPK) found in cardiac muscle; it increases in 4 to 6 hours after chest pain and begins to decline in 12 to 24 hours. LDH-1 increases within 6 to 12 hours after the onset of pain. ESR is nonspecific; it indicates the presence of inflammation or infection. AST increases within the first 12 hours; it is not specific enough to provide a definitive indicator within 4 to 6 hours.

A client who is suspected of having had a silent myocardial infarction has an electrocardiogram (ECG) prescribed by the primary healthcare provider. While the nurse prepares the client for this procedure, the client asks, "Why was this test prescribed?" Which is the best reply by the nurse? 1 "This test will detect your heart sounds." 2 "This test will reflect any heart damage." 3 "This procedure helps us change your heart's rhythm." 4 "The ECG will tell us how much stress your heart can tolerate."

2 Changes in an ECG will reflect the area of the heart that is damaged because of hypoxia. A stethoscope is used to detect heart sounds. Medical interventions, such as cardioversion or cardiac medications, not an ECG, can alter heart rhythm. An ECG will reflect heart rhythm, not change it. Identifying how much stress a heart can tolerate is accomplished through a stress test; this uses an ECG in conjunction with physical exercise.

A client with an abdominal wound infected with methicillin-resistant Staphylococcus aureus (MRSA) is scheduled for a computed tomography (CT) scan of the abdomen. To ensure client and visitor safety during transport, the nurse should implement which precaution? 1 No special precautions are required. 2 Cover the infected site with a dressing. 3 Drape the client with a covering labeled biohazardous. 4 Place a surgical mask on the client.

2 Covering the infected site with a dressing will contain secretions and set up a barrier, thus decreasing the risk for transmission to others. Contact precautions must be used for clients with known or suspected infections transmitted by direct contact or contact with items in the environment. Draping the client with a sheet marked biohazardous does not protect the client's privacy. A wound infected with MRSA can be transmitted to others via contact, not the airborne route; thus a mask is unnecessary.

A nurse instructs a client with a history of frequent urinary tract infections to drink cranberry juice. Which goal is the nurse trying to achieve with this suggestion? 1 Exert a bactericidal effect against the bacteria 2 Prevent bacterial attachment to the bladder wall 3 Improve glomerular filtration rate 4 Relieve the symptoms of dysuria

2 Cranberry juice keeps the bacteria from attaching to the bladder wall. Although bacterial growth may be inhibited, bacteria are not destroyed (bactericidal effect). Glomerular filtration is unaffected by cranberry juice. Cranberry is a preventive manner, not a treatment for dysuria. Cranberry juice may be an irritant in cystitis.

A client is experiencing a myocardial infarction. What should the nurse identify as the primary cause of the pain experienced by a client with a coronary occlusion? 1 Arterial spasm 2 Heart muscle ischemia 3 Blocking of the coronary veins 4 Irritation of nerve endings in the cardiac plexus

2 Ischemia causes tissue injury and the release of chemicals, such as bradykinin, that stimulate sensory nerves and produce pain. Arterial spasm, resulting in tissue hypoxia and pain, is associated with angina pectoris. Arteries, not veins, are involved in the pathology of a myocardial infarction. Tissue injury and pain occur in the myocardium.

A client is admitted to the hospital with a diagnosis of heart failure and acute pulmonary edema. The healthcare provider prescribes furosemide 40 mg intravenous (IV) stat to be repeated in 1 hour. What nursing action will best evaluate the effectiveness of the furosemide in managing the client's condition? 1 Perform daily weights 2 Auscultate breath sounds 3 Monitor intake and output 4 Assess for dependent edema

2 Maintaining adequate gas exchange and minimizing hypoxia with pulmonary edema are critical; therefore, assessing the effectiveness of furosemide therapy as it relates to the respiratory system is most important. Furosemide inhibits the reabsorption of sodium and chloride from the loop of Henle and distal renal tubule, causing diuresis; as diuresis occurs fluid moves out of the vascular compartment, thereby reducing pulmonary edema and the bilateral crackles. Although a liter of fluid weighs approximately 2.2 pounds (1 kilogram) and weight loss will reflect the amount of fluid lost, it will take time before a change in weight can be measured. Although identifying a greater output versus intake indicates the effectiveness of furosemide, it is the client's pulmonary status that is most important with acute pulmonary edema. Although the lessening of a client's dependent edema reflects effectiveness of furosemide therapy, it is the client's improving pulmonary status that is most important.

A client with hypertensive heart disease, who had an acute episode of heart failure, is to be discharged on a regimen of metoprolol and digoxin. What outcome does the nurse anticipate when metoprolol is administered with digoxin? 1 Headaches 2 Bradycardia 3 Hypertension 4 Junctional tachycardia

2 Metoprolol and digoxin both exert a negative chronotropic effect, resulting in a decreased heart rate. Metoprolol reduces, not produces, headaches. These drugs may cause hypotension, not hypertension. These drugs may depress nodal conduction; therefore, junctional tachycardia would be less likely to occur.

Which intervention is most likely to decrease mortality in the septic client? 1 Oxygen 2 Antibiotics 3 Vasopressors 4 Intravenous fluids

2 Of the interventions listed, administering antibiotics is the only intervention that fights the source of the problem. Intravenous fluids, oxygen, and vasopressors are necessary, but are designed to sustain the body until the antibiotic can kill the pathogen.

A 6-month-old infant is brought to the emergency department in severe respiratory distress. A diagnosis of respiratory syncytial virus (RSV) infection is made, and the infant is admitted to the pediatric unit. What should be included in the nursing plan of care? 1 Place in a warm, dry environment. 2 Maintain standard and contact precautions. 3 Administer prescribed antibiotic immediately. 4 Allow parents and siblings to room in with the infant.

2 RSV is highly contagious. The infant should be isolated or placed with other infants with RSV. Standard and contact precautions are instituted to limit the spread of pathogens to others. The infant should receive cool, humidified oxygen by nasal cannula or mask or in a croup tent. Because RSV is extremely contagious, the number of visitors should be limited. Uninfected children should not be allowed near the infant, and as few personnel as possible should care for the infant. Antibiotics are not effective against RSV, and their use is contraindicated.

A nurse is preparing to teach a client to apply a nitroglycerin patch as prophylaxis for angina. Which instruction should the nurse include in the teaching plan? 1 Apply the patch on a distal extremity. 2 Remove a previous patch before applying the next one. 3 Massage the area gently after applying the patch to the skin. 4 Apply a warm compress to the site before attaching the patch.

2 Removing the previous patch before applying the next patch ensures that the client receives just the prescribed dose. Ideally, a patch should be removed after 12 to 14 hours to avoid the development of tolerance. The patch should be rotated among hair-free and scar-free sites; acceptable sites include chest, upper abdomen, proximal anterior thigh, or upper arm. The patch should be gently pressed against the skin to ensure adherence; it should not be massaged. Applying a warm compress to the site before attaching the patch is unnecessary and can result in an excessive absorption of the medication.

When providing dietary instruction to a patient with hypertension, the nurse would advise the patient to restrict intake of which meat? 1 Broiled fish 2 Roasted duck 3 Roasted turkey 4 Baked chicken breast

2 Roasted duck is high in fat, which should be avoided by the patient with hypertension. Weight loss may slow the progress of atherosclerosis and overall cardiovascular disease risk. The other meats are lower in fat and are therefore acceptable in the diet.

The patient informs the nurse that he does not understand how there can be a blockage in the left anterior descending artery (LAD), but there is damage to the right ventricle. What is the best response by the nurse? 1 "The one vessel curves around from the left side to the right ventricle." 2 "The LAD supplies blood to the left side of the heart and part of the right ventricle." 3 "The right ventricle is supplied during systole primarily by the right coronary artery." 4 "It is actually on your right side of the heart, but we call it the left anterior descending vessel."

2 The best response is explaining that the lower portion of the right ventricle receives blood flow from the left anterior descending artery as well as the right coronary artery during diastole.

The nurse supervises an unlicensed assistant personnel (UAP) who is taking the blood pressure of 58-yr-old obese female patient admitted with heart failure. Which action by the UAP will require the nurse to intervene? 1 Waiting 2 minutes after position changes to take orthostatic pressures 2 Deflating the blood pressure cuff at a rate of 8 to 10 mm Hg per second 3 Taking the blood pressure with the patient's arm at the level of the heart 4 Taking a forearm blood pressure because the largest cuff will not fit the patient's upper arm

2 The cuff should be deflated at a rate of 2 to 3 mm Hg per second. The arm should be supported at the level of the heart for accurate blood pressure measurements. Using a cuff that is too small causes a falsely high reading and too large causes a falsely low reading. If the maximum size blood pressure cuff does not fit the upper arm, the forearm may be used. Orthostatic blood pressures should be taken within 1 to 2 minutes of repositioning the patient.

When teaching a patient about dietary management of stage 1 hypertension, which instruction is most appropriate? 1 Increase water intake. 2 Restrict sodium intake. 3 Increase protein intake. 4 Use calcium supplements.

2 The patient should decrease intake of sodium. This will help to control hypertension, which can be aggravated by excessive salt intake, which in turn leads to fluid retention. Protein intake does not affect hypertension. Calcium supplements are not recommended to lower blood pressure.

Sterile warm saline soaks three times a day are prescribed for a client with cellulitis from a puncture wound. The primary nurse places a clean basin, washcloth, and protective pad at the bedside in preparation for the soak but is unable to continue the procedure. What should the nurse assigned to complete the soak do? 1 Continue the procedure as started. 2 Collect new supplies before starting. 3 Discuss the type of soak with the primary healthcare provider. 4 Report the primary nurse to the unit's nurse manager.

2 The supplies at the bedside are not sterile, and the primary healthcare provider prescribed sterile soaks; new supplies must be gathered. Continuing the procedure as started is unsafe; a clean basin and washcloth are not sterile. It is unnecessary to discuss the type of soak with the primary healthcare provider; the primary healthcare provider has already indicated the type of soak desired. Reporting the primary nurse to the unit's nurse manager is not the priority; client safety is the priority at this time.

A patient with aortic valve stenosis is being admitted for valve replacement surgery. Which assessment finding documented by the nurse is indicative of this condition? 1 Pulse deficit 2 Systolic murmur 3 Distended neck veins 4 Splinter hemorrhages

2 The turbulent blood flow across a diseased valve results in a murmur. Aortic stenosis produces a systolic murmur. A pulse deficit indicates a cardiac dysrhythmia, most commonly atrial fibrillation. Distended neck veins may be caused by right-sided heart failure. Splinter hemorrhages occur in patients with infective endocarditis.

Trimethoprim/sulfamethoxazole is prescribed for a child with a urinary tract infection. Which statement by the parent about the drug indicates that the nurse's instructions about administration have been understood? 1 "Mealtime is a good time to give the medication." 2 "I'll make sure to give each pill with 6 to 8 oz (180 to 240 mL) of fluid." 3 "It must be taken with orange juice to ensure acidity of urine." 4 "The drug has to be taken every 4 hours to maintain a blood level."

2 This is a sulfa drug; water must be encouraged to prevent urine crystallization in the kidneys. This drug does not have to be given with meals; it is administered every 12 hours. Orange juice causes an alkaline urine; water is the best fluid to be administered with this drug. This drug maintains the blood level for 8 to 12 hours; it is an intermediate-acting drug.

A patient presents to the emergency department with reports of chest pain for 3 hours. What component of his blood work is most clearly indicative of a myocardial infarction (MI)? 1 CK-MB 2 Troponin 3 Myoglobin 4 C-reactive protein

2 Troponin is the biomarker of choice in the diagnosis of MI, with sensitivity and specificity that exceed those of CK-MB and myoglobin. CRP levels are not used to diagnose acute MI.

When an older client with heart failure is transferred from the emergency department to the medical service, what should the nurse on the unit do first? 1 Interview the client for a health history. 2 Assess the client's heart and lung sounds. 3 Monitor the client's pulse and temperature. 4 Obtain the client's blood specimen for electrolytes.

2 With heart failure, the left ventricle is not functioning effectively, which is evidenced by an increased heart rate and crackles associated with pulmonary edema. The health history interview is done after vital signs and breath sounds are obtained and the client is stabilized. Although an infection would complicate heart failure, there are no signs that indicate this client has an infection. Obtaining the client's blood specimen for electrolytes is inappropriate for immediate monitoring; it should be done after vital signs and clinical assessments have been completed.

An infant is admitted to the pediatric unit with bronchiolitis caused by respiratory syncytial virus (RSV). What interventions are appropriate nursing care for the infant? Select all that apply. 1 Limiting fluid intake 2 Instilling saline nose drops 3 Maintaining contact precautions 4 Suctioning mucus with a bulb syringe 5 Administering warm humidified oxygen

2,3,4 Saline nose drops help clear the nasal passage, which improves breathing and aids the intake of fluids. RSV is contagious; infants with RSV should be isolated from other children, and the number of people visiting or caring for the infant should be limited. Infants with RSV produce copious amounts of mucus, which hinders breathing and feeding; suctioning before meals and at naptime and bedtime provides relief. Fluid intake should be increased; adequate hydration is essential to counter fluid loss. These infants have difficulty nursing and often vomit their feedings. If measures such as suctioning before feeding and instilling saline nose drops are ineffective, intravenous fluid replacement is instituted. The humidified oxygen should be cool. It relieves the dyspnea and hypoxia that is prevalent in infants with RSV.

A nurse is caring for a community-dwelling older adult with hypertension. What interventions should the nurse take to ensure the client's well-being? Select all that apply. 1 Suggest that the client have annual Papanicolaou (Pap) smears and mammograms 2 Promote dietary modifications by using varied techniques 3 Assess the client's current lifestyle and promote lifestyle changes 4 Monitor the client's blood pressure and weight and establish blood pressure screening programs 5 Teach the client about correct body mechanics and the availability of mechanical appliances

2,3,4 When caring for a community-dwelling older adult with hypertension, the nurse should promote dietary modifications, assess a client's current lifestyle and promote lifestyle changes, and monitor the client's blood pressure and weight and establish blood pressure screening programs. When caring for community-dwelling older women with cancer, the nurse should perform annual Papanicolaou (Pap) smears and mammograms for older adults. When caring for a community-dwelling older adult with arthritis, the nurse should teach the client about correct body mechanics and the availability of mechanical appliances.

Which statements related to initial assessment of hypertension by the nurse requires correction? Select all that apply. 1 "Deflating the cuff too slowly will show false-high diastolic readings." 2 "The stethoscope applied too firmly against the antecubital fossa will show a low systolic reading." 3 "If the blood pressure in the left arm is 110/80 mm Hg and in the right arm it is 130/80 mm Hg, it is reportable." 4 "Having the client's arm unsupported while assessing blood pressure will result in a false low reading of blood pressure." 5 "It is normal to have blood pressure of 110/80 mm Hg in the left arm and blood pressure of 120/80 mm Hg in the right arm."

2,4 False low diastolic readings are obtained in clients if the stethoscope is held too firmly against the antecubital fossa. Keeping the arm unsupported while assessing blood pressure results in false high blood pressure values. False high diastolic readings are obtained when the cuff is deflated too slowly. Pressure differences greater than 10 mm Hg between the two arms should be reported because it indicates vascular problems. Normally there is a difference of 5 to 10 mm Hg of blood pressure between the arms.

A client with a long history of cardiovascular problems, including angina and hypertension, is scheduled to have a cardiac catheterization. During preprocedure teaching, what does the nurse explains to the client is the major purpose for catheterization? 1 To obtain the pressures in the heart chambers 2 To determine the existence of congenital heart disease 3 To visualize the disease process in the coronary arteries 4 To measure the oxygen content of various heart chambers

3 Angina usually is caused by narrowing of the coronary arteries; the lumen of the arteries can be assessed by cardiac catheterization. Although pressures can be obtained, they are not the priority for this client; this assessment is appropriate for those with valvular disease. Determining the existence of congenital heart disease is appropriate for infants and young adults with cardiac birth defects. Measuring the oxygen content of various heart chambers is appropriate for infants and young children with suspected septal defects.

What instructions about the use of nitroglycerin should the nurse provide to a client with angina? 1 "Identify when pain occurs, and place two tablets under the tongue." 2 "Place one tablet under the tongue, and swallow another when pain is intense." 3 "Before physical activity, place one tablet under the tongue, and repeat the dose in 5 minutes if pain occurs." 4 "Place one tablet under the tongue when pain occurs, and use an additional tablet after the attack to prevent recurrence."

3 Anginal pain, which can be anticipated during certain activities, may be prevented by dilating the coronary arteries immediately before engaging in the activity. Generally one tablet is administered at a time; doubling the dosage may produce severe hypotension and headache. The sublingual form of nitroglycerin is absorbed directly through the mucous membranes and should not be swallowed. When the pain is relieved, rest generally will prevent its recurrence by reducing oxygen consumption of the myocardium.

The nurse is caring for a patient admitted with chronic obstructive pulmonary disease (COPD), angina, and hypertension. Before administering the prescribed daily dose of atenolol 100 mg PO, the nurse assesses the patient carefully. Which adverse effect is this patient at risk for given the patient's health history? 1 Hypocapnia 2 Tachycardia 3 Bronchospasm 4 Nausea and vomiting

3 Atenolol is a cardioselective β1-adrenergic blocker that reduces blood pressure and could affect the β2-receptors in the lungs with larger doses or with drug accumulation. Although the risk of bronchospasm is less with cardioselective β-blockers than nonselective β-blockers, atenolol should be used cautiously in patients with COPD.

Which intervention would be most beneficial in preventing a catheter-associated urinary tract infection in a postoperative client? 1 Pouring warm water over the perineum 2 Ensuring the patency of the catheter 3 Removing the catheter within 24 hours 4 Cleaning the catheter insertion site

3 Clients who undergo surgery are at a greater risk of acquiring catheter-associated urinary tract infections. Infections can be prevented by removing the catheter within 24 hours, if the client does not need it. Therefore removing the catheter within 24 hours would be the best intervention. While pouring warm water over the perineum helps voiding in the postoperative client and also reduces the chances of infection, this action would not be as beneficial as the former intervention. The catheter should be maintained in its place to avoid leakage and infection. Cleaning the catheter insertion site will definitely reduce the risk of infection, but this action cannot prevent infections if the catheter is inserted for a long time.

A client has been admitted with a urinary tract infection. The nurse receives a urine culture and sensitivity report that reveals the client has vancomycin-resistant enterococci (VRE). After notifying the healthcare provider, which action should the nurse take to decrease the risk of transmission to others? 1 Insert a urinary catheter. 2 Initiate droplet precautions. 3 Move the client to a private room. 4 Use a high-efficiency particulate air (HEPA) respirator during care.

3 Contact precautions are used for clients with known or suspected infections transmitted by direct contact or contact with items in the environment; therefore infectious clients must be placed in a private room. There is no need to insert an indwelling catheter as this can increase the risk for additional infection. Droplet precautions are used for clients known or suspected to have infections transmitted by the droplet route. These infections are caused by organisms in droplets that may travel 3 feet but are not suspended for long periods.

A client has been admitted to the hospital with a diagnosis of methicillin-resistant Staphylococcus aureus (MRSA) in the urine. The client has a urinary catheter in place. No private rooms are available. Which room assignment would be most appropriate for this client? 1 Roommate has a urinary catheter. 2 Roommate is bedridden and uses a bedpan for urination. 3 Roommate has MRSA in the urine. The roommate is ambulatory, but confused. 4 Roommate is alert and oriented with a diagnosis of pneumonia but practices good hygiene when using the bathroom.

3 If a private room is not available, the client may be placed in a room with another client who has the same active infection with the same microorganism if no other infection is present. The roommate with a urinary catheter, who is bedridden, or who has pneumonia does not indicate MRSA in the urine.

A nurse discovers lower extremity pitting edema in a client with right ventricular heart failure. Which information should the nurse consider when planning care? 1 Client has decreased plasma colloid osmotic pressure. 2 Client has increased tissue colloid osmotic pressure. 3 Client has increased plasma hydrostatic pressure. 4 Client has decreased tissue hydrostatic pressure.

3 In right ventricular heart failure, blood backs up in the systemic capillary beds; the increase in plasma hydrostatic pressure shifts fluid from the intravascular compartment to the interstitial spaces, causing edema. Increase in tissue (interstitial) colloid osmotic pressure occurs with crushing injuries or if proteins pathologically shift from the intravascular compartment to the interstitial spaces, pulling fluid and causing edema. In right ventricular heart failure, increased fluid pressure in the intravascular compartment causes fluid to shift to the tissues; the tissue hydrostatic pressure does not decrease. Although a decrease in colloid osmotic (oncotic) pressure can cause edema, it results from lack of protein intake, not increased hydrostatic pressure associated with right ventricular heart failure.

When teaching how lisinopril (Zestril) will help lower the patient's blood pressure, which mechanism of action should the nurse explain? 1 Blocks β-adrenergic effects 2 Relaxes arterial and venous smooth muscle 3 Inhibits conversion of angiotensin I to angiotensin II 4 Reduces sympathetic outflow from central nervous system

3 Lisinopril is an angiotensin-converting enzyme inhibitor that inhibits the conversion of angiotensin I to angiotensin II, which reduces angiotensin II-mediated vasoconstriction and sodium and water retention. β blockers result in vasodilation and decreased heart rate. Direct vasodilators relax arterial and venous smooth muscle. Central acting α-adrenergic antagonists reduce sympathetic outflow from the central nervous system to produce vasodilation and decreased systemic vascular resistance and blood pressure.

A client who is hospitalized after a myocardial infarction asks the nurse why morphine was prescribed. What will the nurse include in the reply? 1 Decreases anxiety and promotes sleep 2 Helps prevent development of atrial fibrillation 3 Relieves pain and reduces cardiac oxygen demand 4 Dilates coronary blood vessels to increase oxygen supply

3 Morphine is a specific central nervous system depressant used to relieve the pain associated with myocardial infarction; it also decreases apprehension and reduces cardiac oxygen demand by decreasing cardiac workload. Dilating coronary blood vessels is not the reason for the use of morphine. Decreasing anxiety and restlessness is not the primary reason for the use of morphine. Lidocaine is given intravenously to prevent fibrillation of the heart.

A nurse is caring for a client who is admitted to the hospital for medical management of heart failure and severe peripheral edema. Which clinical indicator associated with unresolved severe peripheral edema should the nurse initially assess? 1 Proteinemia 2 Contractures 3 Tissue ischemia 4 Thrombus formation

3 Oxygen perfusion is impaired during prolonged edema, leading to tissue ischemia, and should be assessed first. Proteinemia and contractures are not complications resulting from long-term edema. Although thrombus formation may occur, the initial assessment is perfusion (tissue ischemia).

The nurse's physical assessment of a client with heart failure reveals tachypnea and bilateral crackles. Which is the priority nursing intervention? 1 Initiate oxygen therapy 2 Obtain chest x-ray film immediately 3 Place client in a high-Fowler position 4 Assess the client for a pleural friction rub

3 Placing the client in a high-Fowler position promotes lung expansion and gas exchange; it also decreases venous return and cardiac workload. Initiating oxygen therapy may be done, but positioning should be done first because it will have an immediate effect. Time is needed to set up the system for the delivery of oxygen. Maintaining adequate oxygen exchange is the priority; an x-ray film can be obtained, but after breathing is supported. A friction rub is related to inflammation of the pleura, not to heart failure.

To prevent septic shock in the hospitalized client, what should the nurse do? 1 Maintain the client in a normothermic state. 2 Administer blood products to replace fluid losses. 3 Use aseptic technique during all invasive procedures. 4 Keep the critically ill client immobilized to reduce metabolic demands.

3 Septic shock occurs as a result of an uncontrolled infection, which may be prevented by using correct infection control practices. These include aseptic technique during all invasive procedures. Maintaining the client in a normothermic state, administering blood products, and keeping the critically ill client immobilized are not directly related to the prevention of septic shock.

The healthcare provider prescribes a low-fat, 2-gram sodium diet for a client with hypertension. The nurse should explain that the purpose of restricting sodium is to do what? 1 Chemically stimulate the loop of Henle 2 Diminish the thirst response of the client 3 Prevent reabsorption of water in the distal tubules 4 Cause fluid to move toward the interstitial compartment

3 Sodium absorbs water in the kidneys' renal tubules. When dietary intake of sodium is decreased, water is not reabsorbed and edema is reduced. A decrease in sodium will prevent the reabsorption of water. Furosemide stimulates the loop of Henle to inhibit the reabsorption of sodium and chloride at the proximal and distal tubules. Adequate hydration is the major factor that diminishes the thirst response. A low-sodium diet will help move fluid from the interstitial compartment to the intravascular compartment.

A patient with a history of chronic hypertension is being evaluated in the emergency department for a blood pressure of 200/140 mm Hg. Which patient assessment question is the priority? 1 Is the patient pregnant? 2 Does the patient need to urinate? 3 Does the patient have a headache or confusion? 4 Is the patient taking antiseizure medications as prescribed?

3 The nurse's priority assessments include neurologic deficits, retinal damage, heart failure, pulmonary edema, and renal failure. The headache or confusion could be seen with hypertensive encephalopathy from increased cerebral capillary permeability leading to cerebral edema. In addition, headache or confusion could represent signs and symptoms of a hemorrhagic stroke. Pregnancy can lead to secondary hypertension. Needing to urinate and taking antiseizure medication do not support a hypertensive emergency.

The nurse provides discharge teaching to a client with a history of angina. The nurse instructs the client to call for emergency services immediately if the client's pain exhibits which characteristic? 1 Causes mild perspiration 2 Occurs after moderate exercise 3 Continues after rest and nitroglycerin 4 Precipitates discomfort in the arms and jaw

3 When neither rest nor nitroglycerin relieves the pain, the client may be experiencing an acute myocardial infarction. Angina may cause mild diaphoresis; acute myocardial infarction causes profuse diaphoresis, which should be reported. Chest pain after exercise is expected; activity increases cardiac output, which can cause angina. Anginal pain can, and often does, radiate.

A healthcare provider prescribes milrinone for a client with a diagnosis of congestive heart failure who was unresponsive to conventional drug therapy. What is most important for the nurse to do first? 1 Administer the loading dose over 10 minutes. 2 Monitor the ECG continuously for dysrhythmias during infusion. 3 Assess the heart rate and blood pressure continuously during infusion. 4 Have the prescription, dosage calculations, and pump settings checked by a second nurse.

4 Accidental overdose can cause death. Another nurse should verify accuracy of the prescription, dose, and pump settings to prevent harm to the client. Although administering the loading dose over 10 minutes is an appropriate intervention, it is not the first thing the nurse should do. Although monitoring for dysrhythmias is important because they are common with this medication and may be life threatening, it is not the first thing the nurse should do. Although taking the vital signs continuously during the infusion is important because the dose needs be slowed or discontinued if the blood pressure decreases excessively, it is not the first thing the nurse should do.

The blood pressure of an older adult patient admitted with pneumonia is 160/70 mm Hg. What is an age-related change that contributes to this finding? 1 Stenosis of the heart valves 2 Decreased adrenergic sensitivity 3 Increased parasympathetic activity 4 Loss of elasticity in arterial vessels

4 An age-related change that increases the risk of systolic hypertension is a loss of elasticity in the arterial walls. Because of the increasing resistance to flow, pressure is increased within the blood vessel, and hypertension results. Valvular rigidity of aging causes murmurs, and decreased adrenergic sensitivity slows the heart rate. Blood pressure is not raised. Increased parasympathetic activity would slow the heart rate.

A nurse is counseling a woman who has had recurrent urinary tract infections. Which factor should the nurse explain is the reason why women are at a greater risk than men for contracting a urinary tract infection? 1 Altered urinary pH 2 Hormonal secretions 3 Juxtaposition of the bladder 4 Proximity of the urethra to the anus

4 Because a woman's urethra is closer to the anus than a man's, it is at greater risk for becoming contaminated. Urinary pH is within the same range in both men and women. Hormonal secretions have no effect on the development of bladder infections. The position of the bladder is the same in men and women.

An infant with a diagnosis of heart failure is being given furosemide twice a day. Which laboratory value will the nurse report to the healthcare provider? 1 Sodium of 140 mEq/L (140 mmol/L) 2 Ionized calcium of 2.35 mEq/L (1.2 mmol/L) 3 Chloride of 102 mEq/L (102 mmol/L) 4 Potassium of 3.0 mEq/L (3.0 mmol/L)

4 Because furosemide is a potassium-losing diuretic, the potassium should be frequently checked. Normal potassium concentration in infants is 3.5 to 5.0 mEq/L (3.5 to 5.0 mmol/L). A potassium concentration of 3.0 mEq/L (3.0 mmol/L) should be reported to the healthcare provider. The normal range for sodium is 139 to 146 mEq/L (139 to 146 mmol/L), ionized calcium is normally 2.24 to 2.46 mEq/L (1.12 to 1.23 mmol/L), and chloride is normally 98 to 106 mEq/L (98 to 106 mmol/L). The sodium, calcium, and chloride readings are normal.

A 44-yr-old man is diagnosed with hypertension and receives a prescription for benazepril (Lotensin). After providing instruction, which statement by the patient indicates correct understanding? 1 "If I take this medication, I will not need to follow a special diet." 2 "It is normal to have some swelling in my face while taking this medication." 3 "I will need to eat foods such as bananas and potatoes that are high in potassium." 4 "If I develop a dry cough while taking this medication, I should notify my doctor."

4 Benazepril is an angiotensin-converting enzyme inhibitor. The medication inhibits breakdown of bradykinin, which may cause a dry, hacking cough. Other adverse effects include hyperkalemia. Swelling in the face could indicate angioedema and should be reported immediately to the prescriber. Patients taking drug therapy for hypertension should also attempt lifestyle modifications to lower blood pressure such as a reduced-sodium diet.

A client is admitted to the cardiac care unit with a diagnosis of myocardial infarction. The client asks the nurse, "What is causing the pain I am having?" Which explanation of the cause of the pain is the most appropriate response by the nurse? 1 Compression of the heart muscle 2 Release of myocardial isoenzymes 3 Rapid vasodilation of the coronary arteries 4 Inadequate oxygenation of the myocardium

4 Cessation of the blood flow that normally carries oxygen to the myocardium results in pain because of ischemia of myocardial tissue. Myocardial infarction does not involve compression of the heart. The release of myocardial isoenzymes is an indication of myocardial damage; this does not cause myocardial pain. Vasodilation will increase perfusion and contribute to pain relief, not cause myocardial pain.

The nurse provides medication discharge instructions to a client who received a prescription for digoxin following the client's myocardial infarction. Which statement by the client leads the nurse to conclude that the teaching was effective? 1 "I will avoid foods high in potassium." 2 "I must increase my intake of vitamin K." 3 "I should adjust the dosage according to my activities." 4 "It will be important to check my radial pulse rate daily."

4 Checking the radial pulse rate daily is necessary for monitoring cardiac function; digoxin slows and strengthens the heart rate. Digoxin should be withheld and the healthcare provider notified if the pulse rate falls below a predetermined rate (e.g., 60 beats per minute). Hypokalemia increases the potential for digoxin toxicity; potassium intake should be increased, not decreased. An increase in the intake of foods rich in vitamin K is unnecessary; vitamin K is not related to digoxin intake. Adjusting the dosage according to activities is not an appropriate decision for the client; the healthcare provider should make this decision.

A patient is admitted with severe dyspnea, a history of heart failure, and chronic obstructive lung disease. Which diagnostic study would the nurse expect to be elevated if the cause of dyspnea was cardiac related? 1 Serum potassium 2 Serum homocysteine 3 High-density lipoprotein 4 b-type natriuretic peptide (BNP)

4 Elevation of BNP indicates the presence of heart failure. Elevations help to distinguish cardiac versus respiratory causes of dyspnea. Elevated potassium, homocysteine, or HDL levels may indicate increased risk for cardiovascular disorders but do not indicate that cardiac disease is present.

What physiologic alteration does the nurse expect when assessing a 6-month-old infant with bronchiolitis (respiratory syncytial virus [RSV])? 1 Decreased heart rate 2 Intercostal retractions 3 Increased breath sounds 4 Prolonged expiratory phase

4 Infectious and mechanical changes narrow the bronchial passages and make it difficult for air to leave the lungs. As a result of increased respiratory effort and decreased oxygen exchange, tachycardia may develop. Intercostal retractions are unlikely because of overinflation of the chest with air and shallow, rapid breathing. Breath sounds may be diminished because of swelling of the bronchiolar mucosa and filling of the lumina with mucus and exudate.

A pregnant woman at risk of preterm labor is diagnosed with hypertension. Which drug should be administered? 1 Terbutaline 2 Indomethacin 3 Calcium gluconate 4 Magnesium sulfate

4 Magnesium sulfate is used to treat pregnancy induced hypertension. Terbutaline is used to treat preterm labor, but does not reduce hypertension. Indomethacin inhibits prostaglandin activity and seizes uterine contractions. Calcium gluconate is given to reduce magnesium toxicity.

A 67-yr-old woman with hypertension is admitted to the emergency department with a blood pressure of 234/148 mm Hg and was started on nitroprusside (Nitropress). After one hour of treatment, the mean arterial blood pressure (MAP) is 55 mm Hg. Which nursing action is a priority? 1 Start an infusion of 0.9% normal saline at 100 mL/hr. 2 Maintain the current administration rate of the nitroprusside. 3 Request insertion of an arterial line for accurate blood pressure monitoring. 4 Stop the nitroprusside infusion and assess the patient for potential complications.

4 Nitroprusside is a potent vasodilator medication. A blood pressure of 234/118 mm Hg would have a calculated MAP of 177 mm Hg. Subtracting 25% (or 44 mm Hg) = 133 mm Hg. The initial treatment goal is to decrease MAP by no more than 25% within minutes to 1 hour. For this patient, the goal MAP would be approximately 133 mm Hg. Minimal MAP required to perfuse organs is around 60 to 65 mm Hg. Lowering the blood pressure too rapidly may decrease cerebral, coronary, or renal perfusion and could precipitate a stroke, myocardial infarction, or renal failure. The priority is to stop the nitroprusside infusion and then use fluids only if necessary to support restoration of MAP.

Which drug prescribed to a client with a urinary tract infection (UTI) turns urine reddish-orange in color? 1 Amoxicillin 2 Ciprofloxacin 3 Nitrofurantoin 4 Phenazopyridine

4 Phenazopyridine is a topical anesthetic that is used to treat pain or burning sensation associated with urination. It also imparts a characteristic orange or red color to urine. Amoxicillin is a penicillin form that could cause pseudomembranous colitis as a complication; it is not associated with reddish-orange colored urine. Ciprofloxacin is a quinolone antibiotic used for treating UTIs and can cause serious cardiac dysrhythmias and sunburns. It is not, however, responsible for reddish-orange colored urine. Nitrofurantoin is an antimicrobial medication prescribed for UTIs. This drug may affect the kidneys but is not associated with reddish-orange colored urine.


Conjuntos de estudio relacionados

Microeconomics Quiz 2 Study Guide

View Set

Marketing Cloud Administrator - 1

View Set

Macro Chapter 13 Fiscal Policy Review Questions

View Set

Ch. 4 Treatment Settings and Therapeutic Programs

View Set

Chp. 19 Section 19.2-19.5 Dynamic Study Molecule

View Set

Ch 2 Test (Becoming a Professional)

View Set